Космос и астрономия


Ответить в тред Ответить в тред

<<
Назад | Вниз | Каталог | Обновить тред | Автообновление
533 64 116

Тред тупых вопросов №124 Lagrange edition Аноним 09/03/20 Пнд 23:59:18 5528191
Lagrangianpoint[...].jpg (127Кб, 768x512)
768x512
STEREO Visits t[...].mp4 (9190Кб, 1280x720, 00:00:48)
1280x720
Lagrange.jpg (92Кб, 739x1024)
739x1024
InnerSolarSyste[...].png (180Кб, 768x768)
768x768
Тред вопросов о жизни, Вселенной и всём таком.

Спрашиваем то, за что в других местах выдают путёвку в биореактор. Здесь анонимные учёные мирового уровня критически рассмотрят любые гениальные идеи и нарисованные в Paint схемы.

Предыдущий тут: >>550169 (OP)
https://2ch.hk/spc/res/550169.html

Q: Можно быстрее?
A: Можно упасть в пузырь Альбукерке, NASA уже почти надула его.

Q: Я начитался охуительных историй про уфологию, че делать, нам жопа?
A: Да, тебе жопа, можешь сгонять в зогач или куда оттуда пошлют.

Q: Что будет с человеком в вакууме без скафандра / если он упадет на черную дыру / попробует ступить на поверхность газового гиганта/солнца?
A: Он умрёт.

Q: Почему бы не привязать ракету к воздушному шару или стартовать с горы?
A: Космос - это не как высоко, а как быстро, большая часть энергии ракеты уходит на разгон вбок.
Подробнее тут https://what-if.xkcd.com/58/ (английский) https://chtoes.li/orbital-speed/ (перевод)
Аноним 10/03/20 Втр 00:05:35 5528212
Можно ли создать и доставить на поверхность Венеры ровер, который проживёт хотя бы месяц? С учетом текущих технологий естественно.
Аноним 10/03/20 Втр 00:06:43 5528223
>>552819 (OP)
Насколько стабильны орбиты "троянцев" и "греков". Я помню, что точки L4 и L5 являются стабильными, то есть при небольшом отклонении объект "возвращается" на место. Но вот судя по четвертому пику там весьма некислые отклонения от точек есть.
Аноним 10/03/20 Втр 00:12:25 5528234
Аноним 10/03/20 Втр 01:23:11 5528295
Отличный тред для моего вопроса №2, под перекат попал лол.
Я ващет далек от конспиролохии, но посмотрел стыковку Аполлон-11 на орбите Луны и чет не розумию:
Где газ от маневровых? На видео вродь освещенная сторона, какова хуя газа не видно, он че, так сильно рассеивается и вообще свет не отражает?
Где инерция? Какова хуя Орел словно на направляющих меняет ориентацию в пространстве, и не видно вообще инерционных сил? Ну хоть чуть-чуть. Он же небось вручную стыковался, так точно вручную выдать импульс чтобы остановить вращение невозможно. Ну пусть раз, ну пусть два получилось, но не все видео же.
А что за дрожания Орла? Дрожание Колумбии, с которой производится съемка, отсутствуют, да и неоткуда им взяться - во время стыковки им маневров не совершается. А Орел сука дрожит как на тросах.
Вощем что за хуйня, поясните мне. Это реальные съемки или прост потом отсняли документальный материал а-ля "как это было"?
https://www.youtube.com/watch?v=nkUyqe2HM-8
Аноним 10/03/20 Втр 06:03:41 5528596
>>552829
Всё из-за шакального качества видео.
Ты избалован маневровыми сполохами Союза, нужно смотреть какие именно маневровые стоят на аполлонах, какой газ там выпускают, в каком количестве. Давай ты сам поищешь, ок?
На 6:08 от инерции антенна даже двигается с правой стороны модуля.

Вот еще видео с Аполлона 9 где на 12 минуте есть немного кадров стыковки
https://www.youtube.com/watch?v=xytZh161E7o
И вот еще с девятого с 36 минуты, нигде не видно сполохов маневровых даже на земной орбите
https://www.youtube.com/watch?v=HAowocKgOOU
Аноним 10/03/20 Втр 06:12:09 5528607
>>552859
> какие именно маневровые стоят на аполлонах, какой газ там выпускают,
Да обычные гидразин + тетраоксид азота, которые на выходе дают прозрачный азот и прозрачный водяной пар.

Странные движения аппаратов и видимое "отсутствие" инерции на видео объясняются тем, что оно ускорено во много раз, это не реалтайм.
Аноним 10/03/20 Втр 06:40:58 5528618
>>552819 (OP)
Траханное позорище!
Во первых абулькъерро!
Во вторых не надуть, а вытянуть, иначе постановку отменят.
В третих он должен быть очень точен, и до Луны еще далеко.
Аноним 10/03/20 Втр 07:25:58 5528649
>>552859
>На 6:08 от инерции антенна даже двигается с правой стороны модуля.
Я имел в виду движение самого аппарата. Ну то есть при начале маневра вращения относительно одной из плоскостей должен же быть более-менее плавный старт. И более-менее плавная остановка. Даже если маневровыми импульсно работают - с того видео создается навязчивое ощущение что это электродвигатели Орла вращают лол. Но и потом. Допустим что ускоренное воспроизведение объясняет такие резкие движения, но тогда ИРЛ движения были плавными и медленными, и схуяли тогда антенне двигаться. Антенна - это наверное другое, автоматическая ориентация там или еще чего на источник сигнала.
>И вот еще с девятого с 36 минуты
Где на 39 минуте и 40 секунде странные дерганья аппарата. Пожалуй, то что видео ускорено - может объяснить мои вопросы, ну канеш за исключением газа с маневровых. Вот этот >>552860 анон вроде бы прав насчет используемого маневрового топлива вижу какие-то знакомые слова, лол. Но водяной пар должен кристаллизоваться ведь и хоть как-то отражать свет. А хуй. Ууъуъу сьука.
Аноним 10/03/20 Втр 07:49:29 55286610
>>552829
Выхлоп от маневровых ДУ виден далеко не у всех кораблей и далеко не при всех маневрах. И для этого нужен в основном почти контровый свет на фоне неба. Кроме того, если правильно помню, при удержании ориентации (режим ATTH, типа killrot в орбитере или кнопки T в KSP) аполлоновский ECA еще и переключался на импульсно-модулированный режим для клапанов RCS при достижении мертвой зоны угловой скорости, поэтому пшики в финальных этапах сближения всегда мелкие. Могу покопать документацию, но вроде так.
Аноним 10/03/20 Втр 07:53:07 55286711
>>552866
>Могу покопать документацию
Нет, не надо. Мы же не в зогаче в конце концов, а ответы на свои вопросы я более-менее получил, как минимум базу для построения логичного объяснения.
Спасибо вам, спейсаны.
Аноним 10/03/20 Втр 07:55:06 55286812
>>552822
Стабильны-то они стабильны, но это не значит что они тянут к себе что-либо. Эти говна описывают довольно широкие восьмерки и прочие квазиорбитальные фигуры вокруг некоей мнимой точки в системе координат Земли.
Аноним 10/03/20 Втр 08:20:46 55287213
>>552864
>Пожалуй, то что видео ускорено - может объяснить мои вопросы

На 39:34 следи за движениями облаков позади аппарата как они сначала медленно идут, а потом вдруг ускоряются вместе с ускорениями движения аппарата, явно ускоряли.
https://www.youtube.com/watch?v=HAowocKgOOU
Там же кажется вся эта ебучая плёнка, физическая плёнка, которую в катушечках держали в камере, от скорости движения рулона при записи зависел фпс наверное, а потом при оцифровке могли еще при проигрывании с разной скоростью крутить, или же какой-то оцифровщик намерянно решал сократить размер файла чтобы ускорить что-то или сэкономить место
Аноним 10/03/20 Втр 08:22:21 55287414
Animationof2010[...].gif (7850Кб, 560x420)
560x420
Аноним 10/03/20 Втр 12:42:31 55289715
>>552821
>Можно ли создать и доставить на поверхность Венеры ровер, который проживёт хотя бы месяц
Можно
>С учетом текущих технологий
Нет. Современные технологии долго и упорно ломились в одну узкую нишу и вне её слабо применимы. Разработка устройств ля экстремальных условий происходит почти с нуля чаще всего.
10/03/20 Втр 12:43:04 55289816
>>552829
>Где газ
Вокруг посмотри
Аноним 10/03/20 Втр 13:37:23 55290717
>>552874
Спасибо. Понятно, что точки сами по себе ничего не тянут, там же массы нет, просто за счет комбинации гравитации двух тел создается некое движение в окрестности этих точек. Я просто не ожидал, что окрестность такая большая.
Аноним 10/03/20 Втр 17:10:01 55292518
>>552874
А где можно как на пике с телами играться и залипать на их траектории как там вот?
Аноним 10/03/20 Втр 17:22:31 55292819
>>552925
Прям "как там вот" - в GMAT
https://sourceforge.net/projects/gmat/
Пиздатейший симулятор, рекомендую. Но для космических аппаратов. Для астрономических объектов есть Universe Sandbox, но вообще думаю что конкретно эта гифка была нарисована в питоне наверняка, при помощи одного из астродинамических пакетов, или вообще по формуле.
Аноним 10/03/20 Втр 21:41:57 55298120
Аноним 11/03/20 Срд 13:50:12 55302621
Аноним 11/03/20 Срд 14:06:35 55302822
>>552819 (OP)
Ебанный стыд...
Во-первых, Алькубьерре.
Во-вторых, не упасть, а создавать вокруг корабля изнутри (иначе кина не будет).
В-третьих, НАСА искривляет пространство на десятимиллионную часть, контролируя это сверхточными интерферометрами, до самого варп-привода здесь - как до Антарктиды раком.
Аноним 11/03/20 Срд 14:25:45 55303123
>>553026

Так чё? Вселенная таки замкнутая? Да?
Аноним 11/03/20 Срд 19:34:30 55304624
Аноним 12/03/20 Чтв 02:44:49 55306125
>>552874
Объясните почему переодически колбасит землю? Кривизна орбиты? Но тогда почему это происходит не всё время, а с перерывами? циклы миланковича же десятки тысяч лет а не столетие полтора.
Аноним 12/03/20 Чтв 08:42:46 55307526
Если 99.9999% топлива ракеты расходуется на набор орбитальной скорости, то зачем нужны запуски ракет с самолётов и проекты по типу катапульт и пушек? Для одноступенчатых ракет? Для запуска без стартового стола? Потому что с 20-30 км легче стартовать?
Аноним 12/03/20 Чтв 09:14:15 55307627
Аноним 12/03/20 Чтв 09:29:57 55307728
>>553061
>Объясните почему переодически колбасит землю?
В смысле "колбасит"?

>>553075
>зачем нужны запуски ракет с самолётов и проекты по типу катапульт и пушек?
Катапульт и пушек НЕ ТУ ТВЕРДО И ЧЕТКО. На луне может быть и возможно запилить такой старт, на земляшке - нет.
С самолетов т.к. можно на любое наклонение и на большом расстоянии от космической инфраструктуры стартовать. Под окно гадать сильно не надо, чтоб космодром четко в нужном месте был, хочешь сегодня пулять - можно 200км туда-то отлететь и стартовать. На следующей неделе? 200км в другую сторону и пуляй.
Основной профит в вариантах направления старта.
С мыса Карнавала пулять можно только на восток, а с Вварденфелла на сервер, грубо говоря, а с самолета OTLETEL PODALSHE и пульнул нахуй.
Аноним 12/03/20 Чтв 10:37:07 55307929
>>553046
По мне так думается, что это самый лучший вариант.

Чёрный шарик снаружи определённого размера, который может и расти и сжиматься, а внутри как в игре при выходе за предельные значения некого пространства элемент переносится, а по факту просто отображается в другом месте.
Аноним 12/03/20 Чтв 12:35:21 55308230
>>553077
>С мыса Карнавала пулять можно только на восток, а с Вварденфелла на сервер, грубо говоря, а с самолета OTLETEL PODALSHE
Если ты действительно борешься за эффективность, то на полярные орбиты будешь запускать с самолёта, летящего на широте Варденфелла, а на восток с экватора. Неизбежно.
Аноним 12/03/20 Чтв 16:25:13 55309831
>>553077
>В смысле "колбасит"?
На изображении синяя точка дёргается из стороны в сторону.
Можно предположить, что это кривизна орбиты синей точки. Однако она не всегда двигается туда-сюда, а с перерывами.
К чему бы это?
Аноним 12/03/20 Чтв 17:23:42 55310132
>>553061
>>553098
Ты посмотри на масштаб лет на гифке. У Земли орбита не круговая и возмущенная, трястись будет в любом случае.
Аноним 12/03/20 Чтв 17:25:39 55310233
>>553075
>зачем нужны запуски ракет с самолётов
Ради мобильности точки пуска, ради запуска без стартового стола. И да, поскольку масса ракеты растёт экспоненциально от дельты, приличная доля массы экономится таки.
Аноним 12/03/20 Чтв 17:37:56 55310334
>>553101
Я слышал только о циклах миланковича, которые происходят с периодом десятки и сотни тысяч лет, а не просто сотни. Что это за возмущения такие?
Аноним 12/03/20 Чтв 17:55:27 55310635
>>553103
Какие еще циклы миланковича, причем тут они? Циклы Миланковича относятся к инсоляции, они никак не влияют на орбиту. Они вызваны её характеристиками - периодичностью параметров, гравитационными возмущениями и т.п.

>Что это за возмущения такие?
Гравитационные возмущения орбиты, вызванные притяжением других тел, особенно Юпитера. Это только в учебниках тела летают по эллипсам, на практике тел дохуя и они тянут друг друга в разные стороны. Строго говоря, никто не знает даже, стабильна ли орбита Земли, да и Солнечная система вообще, дальше нескольких десятков миллионов лет вперед.
Аноним 12/03/20 Чтв 18:04:52 55311036
>>553106
Вот не надо этого циклы миланковича это изменение экстриситета под действием в основном юпитера, в следствии которого происходит изменение инсоляции, а не просто изменение инсоляции с нихуя.
Что-то мне не верится, что есть какой-то периодический процесс раз в 100 лет, относящийся к земле, и который не исследовали уже вдоль и поперёк.
Аноним 12/03/20 Чтв 18:12:16 55311337
image.png (104Кб, 447x246)
447x246
что мешает взять углерод и облучив его альфа и бета излучением получить кислород?
что мешает притащить на марс астеройд из углерода и превратить его в кислород?
Аноним 12/03/20 Чтв 18:13:46 55311538
>>553110
>Вот не надо этого циклы миланковича это изменение экстриситета
>Milankovitch cycles describe the collective effects of changes in the Earth's movements on its climate
По определению. Почитай, что именно открыл Миланкович, кто он был и почему это назвали его именем.
>изменение экстриситета под действием в основном юпитера
Это причина этих циклов. Об этом и речь.

>Что-то мне не верится, что есть какой-то периодический процесс раз в 100 лет, относящийся к земле, и который не исследовали уже вдоль и поперёк.
С чего ты решил, что он есть? Если ты про трясущуюся Землю на гифке - это резонанс эллиптичности орбиты и периода кадров, возможно еще каких-то процессов, учтенных конкретно в той симуляции. Но главным образом биения дискретизации кадров гифки.
Аноним 12/03/20 Чтв 18:19:25 55311639
>>553113
Мешает требуемая энергия, количество полученного, радиоактивность полученного изотопа
Если ты сможешь CN-цикл поддерживать, у тебя никаких проблем не будет с доставкой кислородия
Аноним 12/03/20 Чтв 18:55:10 55312340
image.png (20Кб, 782x759)
782x759
что мешает взять и привезти на орбиту земли по частям огроменнейший космический танкер, накачать его до отвала воздухом земли, опустив с орбиты шланг, а потом с помощью луны и троса запустить себя на марс с ++++ скоростью так, чтобы танкер разбился и освободил кислород? или сделал гравитационный маневр, сбросив скорость хз. принцип ускорения на картинке как объяснить хз.
Аноним 12/03/20 Чтв 19:02:47 55312441
>>553123
Да ничего не мешает, просто один такой это будет пук в никуда.
Для масштабов целого Марса - мешает гигантский объем перевозимого и ограниченность земной атмосферы. Ну и по мелочи, отсутствие цикла, восполняющего/сохраняющего кислород.
Аноним 12/03/20 Чтв 19:11:52 55312542
>>553124
>>гигантский объем перевозимого
да большой, но зато есть невесомость. можно сделать большой пакет, который раздуется к примеру. можно сжать. можно просто строить ооочень большой корабль. ты не ограничен.
>>ограниченность земной атмосферы
марс меньше земли. можно по братски поделить напополам атмосферу. может хватит для жизни растений, а это уже пол дела
>>отсутствие цикла, восполняющего/сохраняющего кислород.
можно все засадить там растениями, животными и вот тебе цикл.
Аноним 12/03/20 Чтв 19:16:27 55312643
>>553123
1.Трос надо делать из неведомого вещества выдерживающий собственный вес.
2.Сам трос имеет массу, которую надо запустить в космос. И что-то мне кажется что лучше уж сразу кислород запускать.
3.Трос имеет площадь поверхности большей чем площадь поверхности самого корабля. Соответственно трос затормозится, начнёт падать на землю с кораблём или без . И скорее всего сгорит от трения.
4. трос длинной 380 000 км не выдержит даже собственный вес какими бы футуристичными материалы не были. 35 000 уже считается фантастикой. Трос 380 000 из нанотрубок будет толщиной с видимую часть вселенной.
5 Трос не зацепится за луну, ну или надо будет прокопать всю луну до мантии и сделать из него супер крюк что не отвалится под силой этого танкера. Опять же легче на ракетах.
6.Скорость вращения луны недостаточна дабы добратся до марса ракеты всё равно понадобятся для доускорения и по прилёту на торможение.
7.Ну долетишь ты до марса. Танкер разобьётся и выпустит одну квинтилионную долю атмосферного воздуха, который даже не кислород, а азото-кислородная смесь.
8.Ну привёз ты квинтилион танкеров с атмосферой земли. Углекислый газ атмосферы марса всё так же будет тебя удушать и дышать ты там не сможешь.
9. И вот ты привёз секстиллионы танкеров на марс. Хорошо на марсе жить можно. А на земле уже нет.
>>553125
Атмосферное давление слышал, не? по братски поделишься и на земле будут только мхи да лишайники высокогорья. Атмосфера как бы не увеличивается от растений, они просто берут CO2 и забирают C, а потом это C превращается обратно в CO2 под действием разлагающих бактерий и грибов. объём воздуха не увеличивается.
Аноним 12/03/20 Чтв 19:19:44 55312744
>>553098
У земли афелий 152 гигаметра, перигелий 147 гигаметра.
Каждый год земля на 5 миллионов километров то ближе, то дальше к солнцу.
Аноним 12/03/20 Чтв 19:27:45 55312845
>>553126
ты меня наверное не понял, но и корабль, и трос будут в космосе, а значит никакого трения, веса и тд не будет. Зацепиться можно будет обогнув луну в лассо как у ковбоев)) да и не одну квинтилионную можно перевезти. объемы корабля не ограничены особо + можно сжимать воздух.
а на счет выработки кислорода. я только что читал, что фотосинтез это свет + вода = кислород. растениям нужна только вода и будет пополнение атмосферы в полноценную и неясно чем тебе не нравится азото-кислородная смесь? как будто ты чем то другим сейчас дышишь. ну а на землю пофиг (упадет на 20-30% процентов да на эверест не залезть, пару миллионов слабачков помрут, но вскоре она должна восстановится потому что я писал выше.
Аноним 12/03/20 Чтв 19:28:06 55312946
>>553123
>что мешает взять и привезти на орбиту земли по частям огроменнейший космический танкер,
Цена, отсутствие инфраструктуры позволяющей это делать в разумные сроки.
>накачать его до отвала воздухом земли, опустив с орбиты шланг
1. Шланг большой длины будет весить как сам танкер, даже если из нанотрубок
2. На орбитальной скорости его порвет + он сгорит в атмосфере
3. Он представляет собой охуенный тормоз, сведя танкер с орбиты в кратчайшие сроки
4. Нельзя качать насосом находящимся сверху выше определенного предела: нельзя качать атмосферу из космоса, нельзя качать воду из трубы высотой выше 10 метров, нельзя качать ртуть из трубки высотой выше 780 миллиметров.
И многое другое.
>, а потом с помощью луны и троса запустить себя на марс с ++++ скоростью так, чтобы танкер разбился и освободил кислород?
Прочность троса, опять же, достаточно слингшота, но танкер в любом случае должен быть управляемый, иначе промахнется без коррекций на полпути.
Но самое главное - попадание йобы в марс на скорости выше скорости убегания просто выкинет обломки и кучу газа с поверхности нахуй. Плюс, кислород окислит все что только можно.
Плюс для создания атмосферы танкер должен быть размером с планетоид и единовременное столкновение с такой массой будет катаклизмом.
Аноним 12/03/20 Чтв 19:38:03 55313047
>>553126
Кек, ты так же ответил.

>>553128
>корабль, и трос будут в космосе, а значит никакого трения, веса и тд не будет
Ты говорил шланг. Который опускал в атмосферу. Который является охуенным тормозом.
Будет еще какое.
Кстати, еще момент, который мы упустили - огромный "танкер" посбивает спутники своим телом и должен быть дохуя прочным, т.к. из-за многокилометровых размеров будет испытывать заметные приливные силы.
>>553128
>а на счет выработки кислорода. я только что читал, что фотосинтез это свет + вода = кислород.
Нет, это некоторые бактерии воду расщепляют. Растения поглощают СО2, используя С для роста и выделяя кислород. Днем, во время фотосинтеза. Ночью они полученные днем сахара сжигают так же как и аэробные организмы.
>растениям нужна только вода и будет пополнение атмосферы в полноценную
А еще давление, температура, многочисленные параметры грунта и симбиоз с микроорганизмами.
>и неясно чем тебе не нравится азото-кислородная смесь?
Он о том, что ты чистого кислорода не везешь, как ты писал в оригинальном посте.
>ну а на землю пофиг (упадет на 20-30% процентов да на эверест не залезть, пару миллионов слабачков помрут, но вскоре она должна восстановится потому что я писал выше.
Не восстановится. Атмосфера ниоткуда не берется, она есть и потихоньку сдувается. Пополняться атмосфера сама может только за счет геологической активности, вулканизма, и основной газ который выходит в таких случаях - you guessed it - CO2.

Если ты собрался на Марсе атмосферу запиливать - ебашь водно-аммиачными кометами по нему. Программа эта на много тысяч лет. Венеру можешь заодно поколбасить.
Аноним 12/03/20 Чтв 19:48:43 55313148
image.png (93Кб, 307x166)
307x166
Аноним 12/03/20 Чтв 19:54:50 55313349
>>553131
Не парься, Илон, астероидные ISRU и долговременные хабитаты тебе помогут продержаться пока марс будешь терраформировать.
Аноним 12/03/20 Чтв 22:58:57 55314650
>>553110
Время Ляпунова применимо к Земле так же как и к другим объектам. А у Земли оно еще и точно не известно. То есть все еще хуже
Аноним 12/03/20 Чтв 23:18:35 55314951
43.jpg (23Кб, 450x450)
450x450
Почему на Луне такие покатые холмы?
Атмосферы нет, гравитация ниже. Из-за чего порода эррозирует?
Аноним 13/03/20 Птн 00:17:09 55315552
>>553149
Эрозия с одной "р".
И глагол "эродировать" от "erode" - https://ru.wiktionary.org/wiki/эродировать

Холмы как раз и будут рассыпчатыми и покатыми в отсутствие эрозии, анон. Каньоны и резкие уступы созданы на нашей планете благодаря эрозии, а не вопреки ей. Ещё один механизм мог бы быть - живая и очень активная тектоника, которой у луны не пахнет.
Потому весь рельеф образован тем, что отсыпется от взрыва. Поверхность луны суть - набор кратеров.
Аноним 13/03/20 Птн 00:53:55 55316153
>>553149
это горы очень колючей твёрдой пыли. мы привыкли что пыль мягкая или мелкая, но там это просто колючий толчёный камень, который миллионами лет падал сверху от ударов и выбросов породы.
Аноним 13/03/20 Птн 01:04:39 55316454
Аноним 13/03/20 Птн 01:07:44 55316555
>>553164
Может будет, а может нет. Раньше люди думали что можно за край Земли выпасть.
Аноним 13/03/20 Птн 01:11:00 55316656
>>553164
Не будет, человечества не будет, не переживай.
Аноним 13/03/20 Птн 04:39:18 55318257
>>553164
Всегда мне казалась это видео каким-то сектанским.
Аноним 13/03/20 Птн 06:18:15 55318858
>>553164
Не ссы, ты всё равно умрёшь гораздо раньше, эти проблемы уже не будут твоими. Пусть постхьюманы себе этим мозги ебут.
Аноним 13/03/20 Птн 08:16:02 55319059
>>553164
Во-первых, наша космология даже в рамках одной галактики пока полна дыр и условностей, а в масштабах Вселенной это вообще гадание на кофейной гуще. Мы даже с расширением пока ещё окончательно не определились, есть ли оно глобально вообще, ускоренное ли оно, и сколько там на самом деле производных.

Во-вторых, тепловая смерть вселенной - процесс ассимптотический. Даже если разность потенциалов крайне мала - она никогда не равна нулю, и динамические самоорганизующиеся системы продолжат существовать. Допустим, звёзды давно выгорели, всё остыло, и вселенная заполнена ошмётками чёрных карликов температурой 0.0001К, совершающих одно малюсенькое колебание за миллиард лет. Это для нас миллиард лет. А для них это микросекунда, и никакой тепловой смерти они даже не заметили. Например, если сравнивать с кварковой эпохой, то тепловая смерть вселенной давно произошла. Может там даже зверушки какие-то жили на неизвестных физических принципах, чьи сотни поколений умещались в одну нашу микросекунду.

В-третьих, есть такой прикол, как время возврата Пуанкаре. И хотя он оперирует ОЧЕНЬ большими масштабами времени, это такое неизбежное математическое следствие существования мира. Когда-нибудь через N лет (где N всё ещё намного меньше числа Грэма) ты, анон, снова родишься и будешь писать на спейсач. А потом снова родишься. И во всех возможных вариациях.

Ну и в-четвёртых, есть старое доброе эвереттовское самоубийство, которое можно понимать расширенно. Если где-то есть наблюдатель, то совершенно похуй, где и из чего сделано это самое "где", будь оно хоть параллельным миром, хоть тем же миром через время возврата Пуанкаре, хоть пузырём на соседней бране, хоть результатом симуляции. Наблюдатель обречён существовать, поскольку вариантов, где он перестал существовать, он пронаблюдать не способен.

Короче, не рефлексируй по всякой хуите. Поважнее дела есть.
Аноним 13/03/20 Птн 12:01:23 55320060
Аноним 13/03/20 Птн 14:55:02 55320861
Аноним 13/03/20 Птн 15:00:46 55320962
>>553190
>Во-первых, наша космология даже в рамках одной галактики пока полна дыр и условностей
А каких именно?
Аноним 13/03/20 Птн 15:30:10 55321163
>>553149
>Из-за чего порода эррозирует?
Эродирует. Из-за постоянных ударов метеоритов образуются осколки и пыль. А поскольку у Луны все же заметно ненулевое тяготение, значительная часть падает вокруг, а не улетает в ебеня. Из-за солнечного воздействия: днем (лунным) порода нагревается, ночью остывает, что приводит к растрескиванию и измельчению.
Аноним 13/03/20 Птн 15:58:50 55321364
>>553164
Какая разница, все равно помрем раньше из-за короновируса. В живых останутся только хикке-сычи, а они не размножаются.
Аноним 13/03/20 Птн 16:21:55 55322165
>>553209
Проблема вращения галактик, из которой родился ненаблюдаемый костыль в виде тёмной материи и целая пачка альтернативных теорий гравитации типа MOND,TeVeS и.т.п. Дальше пошли всякие упоротые экстраполяции и экстраполяции экстраполяций, которые пока ни проверить, ни опровергнуть. Есть даже такие версии, что вся наблюдаемая Вселенная находится внутри охренительно большой чёрной дыры, и целые пачки далеко идущих выводов. Вплоть до космологических червоточин и N>3-мерной вселенной за пределами горизонта событий, который сплющил всё лишнее в планковский размер.

>>553200
Вот это одна из таких экстраполяций, порождённых другим костылём, которую распиарили, потому что она выглядит страшно и зловеще. Почти как распад ложного вакуума и гравитационный антивзрыв великого аттрактора. Такая-то космологическая крипипаста.
По факту у нас большие проблемы даже с оценкой скорости далёких объектов. Мы отлично можем измерить скорость "на нас" и "от нас" допплеровским способом, но у нас большие проблемы с измерением скорости вбок.

Например, мы запросто можем оказаться в квазистационарной неравномерно-пульсирующей вселенной, в которой различные области пространства (размерами в миллиарды световых лет) попеременно сжимаются и расширяются. Находясь в единственной точке и в единственной фазе мы вообще ничего не можем сказать наверняка, а телепорт в другие нам пока не завезли.
Аноним 13/03/20 Птн 18:11:06 55323066
>>553221
>ненаблюдаемый костыль
Интеллект твой ненаблюдаем. Проследуй в зогач.
Аноним 13/03/20 Птн 18:37:55 55323167
>>553221
Есть кривая скорости расширения в разные возраста вселенной. Наука не стоит на месте.
Аноним 13/03/20 Птн 19:16:57 55323368
2gExnXvnpoU.jpg (32Кб, 960x704)
960x704
>>553230
Звоните в нобелевский комитет, двачер обнаружил тёмную материю у себя в очке.

>>553231
Если ты про пикрелейтед, то это всего лишь обоснование радиального движения пространства, а не радиального разлёта галактик по неподвижному пространству. Про тангенциальные скорости она вообще ничего не говорит. Да и гарантий, что постоянная Хаббла на самом деле одномоментно-одинакова во всех точках наблюдаемой вселенной, не даёт. Мы вообще не в состоянии наблюдать то, что там в данный момент происходит.
Аноним 13/03/20 Птн 19:51:41 55323669
>>553233
Я оговорился Есть кривая ускорения вселенной(хотя в общем-то одно и тоже). Очевидно мы не в центре вселенной, и потому скорость убегания конкретной области от нас так же велика как скорость убегания той точки от другой точки с такого-же расстояния. А галактики не очень-то и быстры относительно процессов крупномасштабной структуры вселенной. И скорости эти мерят по сверхскоплениям, а не по отдельным галактикам. И в общем-то зная среднюю скорость компонентов , чисто математически получаем скорость сверхскопления относительно нас, с минимальной погрешностью, которые считай не двигаются. И на их доплеровское смещение влияет расширение, а не гравитация, которая на таких масштабах ничтожна, если вообще существует.

Да и галлактики с нихуя не разлетаются хотя бы потому, что нет центра из которого надо разлетаться.
Аноним 13/03/20 Птн 22:20:59 55324670
Насколько было бы выгодней запускать небольшие космические аппараты не с земли, а со стратостатов? Скажем с 30 километровой высоты.
Аноним 13/03/20 Птн 22:55:28 55325171
>>553246
Категория: Платиновые вопросы ТТВ
Посмотри ответ в самом низу шапки треда.
Аноним 14/03/20 Суб 03:19:38 55329072
Аноним 14/03/20 Суб 06:49:09 55329673
>>553251
Я видел, но нормального ответа там нет.
Во-первых, в вакууме жрд работают намного эффективнее, чем при атмосферном давлении.
Во-вторых, больше высота - меньше вес, больше удельная весовая тяга.
В-третьих, значительно меньше аэродинамических потерь.
В-четвёртых, можно сразу стартовать под меньшим углом к горизонту, и с бОльшим ускорением.
В итоге общая дельта, требуемая для выхода на орбиту, должна быть меньше чем при старте с поверхности.
Вопрос на сколько, примерно?
Аноним 14/03/20 Суб 07:05:01 55329774
>>553296
>Во-первых, в вакууме жрд работают намного эффективнее, чем при атмосферном давлении.
Неправда. 300 против 330 секунд обычно, т.е. 10% в лучшем случае.
>Во-вторых, больше высота - меньше вес, больше удельная весовая тяга.
Неверно. Ты разгоняешь не вес, а массу. Масса не меняется.
И даже если типа выше легче - на 400км, высота МКС, вес составляет 90% от веса на поверхности.
>В-третьих, значительно меньше аэродинамических потерь.
Верно. Это один из плюсов. Сыграет ли он против минусов такого подхода?
>В-четвёртых, можно сразу стартовать под меньшим углом к горизонту, и с бОльшим ускорением.
Второй плюс, но про ускорение хуита.
>В итоге общая дельта, требуемая для выхода на орбиту, должна быть меньше чем при старте с поверхности.
Где-то на процентов десять в самом лучшем случае.
>Вопрос на сколько, примерно?
Процентов десять.
И это в самом идеальном случае. Факторов дохуища, и все эти бонусы от высокого старта нивелируются огромной кучей минусов воздушного старта.
Аноним 14/03/20 Суб 07:19:02 55329875
Почему восход солнца ровно за моей спиной и прямо передо мной луна которая освящена лишь на половину? Она должна быть целиком сетиться а не напополам.
Как это работает?
Вот нихуя не работает и потом удивляетесь почему люди выбирают плоскую землю чем ваш пиздежь
Аноним 14/03/20 Суб 07:40:33 55329976
>>553296>>553246
Если запускать ракету со статично висящего аэростата, то экономишь только на аэродинамическом сопротивлении и небольшом начальном бусте к УИ. Стартовать в любом случае придётся вверх, ибо сразу ты горизонтальную скорость для инерционного "висения" над убегающей поверхностью не наберёшь. Топлива реально потребуется меньше, но ненамного. Процентов на 5-10. Т.е. тебе в любом случае придётся тащить в стратосферу несколько сотен тонн. Масшатабирование существующих стратостатов даёт требуемый объём баллона примерно в 0.25-0.3 кубических километра!

Короче, идея - полная хуита. Можно заменить стратостат на очень жирный самолёт - это уже будет технически воплотимо, но всё равно экономия маленькая. А вот если взять большой и охуенно быстрый самолёт, хотя бы на 5-6М, то будет уже 2..4 кратное уменьшение как требуемого топлива, так и требуемой мощности маршевого двигателя.
Аноним 14/03/20 Суб 08:51:00 55330077
>>553297
>Неправда. 300 против 330 секунд обычно, т.е. 10% в лучшем случае.

Справедливости ради, 330 против 300 это с соплом, оптимизированным под давление на уровне моря, с 30 км можно будет стартовать сразу с гораздо более широким вакуумным соплом. Для примера цифры от спейсиксовского Мерлина-Д:

С атмосферным соплом со степенью расширения 16:
- на уровне моря: 282 с,
- в пустоте: 311 с.

С вакуумным соплом со степенью расширения 165:
- в пустоте: 348 с, т.е. разница уже не 10, а 23%.
Аноним 14/03/20 Суб 08:52:42 55330178
>>553300
Соглашусь, выгода есть, и я не говорил что ее нет.
Но гемор с воздушным стартом все равно того не стоит.
Аноним 14/03/20 Суб 09:11:36 55330479
>>553298
Что надо сделать что-бы луна была освещена полностью. Надо находится +- ровно между солнцем и луной. И либо ты улетаешь в космос и наблюдаешь там полную луну, либо ждёшь когда луна окажется за землёй на которой ты находишься. Т.е. на ночной стороне. В это время луна будет полной, но солнца ты не увидишь ибо ночь.
И вот ты наблюдаешь солнце на восходе\закате. Проведи линию от солнца через себя и обнаруж, что для того что бы видить полную луну ты должен смотреть за горизонт. Либо солнце выше горизонта либо луна. Когда видна и луна и солнце, то соответственно ты не находишься между луной и солнцем.
И т.к ты скорее всего не на экваторе и солнце у тебя не встаёт ровно на востоке и не садится ровно на западе, в то время как луна летит по своей орбите с не нулевым наклонением, то скорее всего не на одной линии и луна на каком нибудь запада-запада-юге, а солнце на каком-нибудь юго-востоке.
Прежде чем опровергать круглую землю следует опровергнуть геометрию.
Аноним 14/03/20 Суб 09:24:43 55330580
>>553304
А почему луна всегда вертикально освещена? Почему нет чтобы луна под углом освещена была, всегда четко сверхувниз граница?
Аноним 14/03/20 Суб 12:05:32 55331381
мавритания.png (107Кб, 320x512)
320x512
>>553305
На экваторе линия терминатора, т.е. граница света и тени проходит слева на право. И в зависимости от твоего расположения будет разный угол относительно поверхности. Например: Петербург находится на 60 широте. В момент равноденствия, в то время когда луна находится в узле своей орбиты, угол терминатора и земной поверхности будет повёрнут на 60 градусов. Соответственно летом будет уменьшатся и становится более острым, и когда луна будет в перигее и апогее, то увеличиватся то уменьшатся. В первом случае с амплитудой 23.5 градуса и лунная орбита что-то около 5 градусов.
А почему тебе кажется что луна всегда ровно светит мне неведомо. Возможно по той же причине по которой перевёрнутый вверх ногами или набок текст не осознаётся как перевёрнутый. И вообще это европоцентризм думать что сверху вниз, а не слева на право. Вот тебе флаг Мавритании с изображением луны.
Аноним 14/03/20 Суб 14:40:40 55332882
Вопрос был про вывод совсем небольших спутников на опорную орбиту. Максимум сотня килограмм полезной нагрузки.
>>553297
>про ускорение хуита
Раз не надо преодолевать сопротивление воздуха, "лишняя" прочность позволит более высокую перегрузку.
>Процентов десять
Это немало так-то.
>>553299
>Т.е. тебе в любом случае придётся тащить в стратосферу несколько сотен тонн
Не надо сотни тонн, вопрос был про маленькие аппараты.
Стартовая масса будет несколько тонн (ну пусть десять тонн).
>очень жирный самолёт
Стратостат вменяемых размеров будет, вероятно, в сотни раз дешевле такого самолёта.
>>553300
>т.е. разница уже не 10, а 23%
Уже не так уж и мало, не правда ли?
Аноним 14/03/20 Суб 14:59:34 55333083
>>553328
> Вопрос был про вывод совсем небольших спутников на опорную орбиту. Максимум сотня килограмм полезной нагрузки.
Гугли Pegasus и Launcher One. 300-400 кг, страт с 747. Или, лол, загугли zero2infinity, те ещё омичи
Аноним 14/03/20 Суб 16:36:29 55334184
>>553328
>Раз не надо преодолевать сопротивление воздуха, "лишняя" прочность позволит более высокую перегрузку.
Сопротивление воздуха имеет значение только для обтекателя, плюс перегрузка не является важным ограничивающим фактором для ракет. Ничего не мешает ракетам стартовать хоть с ускорением в 10g, просто ставить такие мощные двигатели тупо невыгодно с точки зрения максимизации полезной нагрузки, они слишком много весят. Любая вменяемая орбитальная ракета будет взлетать хоть с уровня моря, хоть с горы, хоть со стратостата с TWR в пределах 1.2~2.

>Это немало так-то.
Это практически нихуя по сравнению с дополнительной сложностью воздушного старта и невозможностью использовать более энергетические криогенные топлива.

>Стратостат вменяемых размеров
Стратостат вменяемых размеров это оксюморон, из-за ничтожной плотности воздуха и, соответственно, ничтожной подъемной силы газа на таких высотах в любом случае требуются шары огромного объема. Тот же Red Bull Stratos поднимал около тонны веса при объеме в 850 тысяч кубических метров, у тебя даже с небольшой ракетой выйдет шар под десяток миллионов кубометров.

>т.е. разница уже не 10, а 23%
Это разница в удельном импульсе только на начальном этапе работы первой ступени, уже через минуту после старта ракета в любом случае будет лететь в практически полном вакууме. Разница в дельте будет гораздо меньше. Хотя возможность использовать вакуумное сопло для первой ступени это неплохой плюс, да.
Аноним 14/03/20 Суб 17:31:35 55335285
Анон, а вот есть обсерватория DSCOVR
>наблюдает нашу Землю с расстояния 1,5 млн км, располагаясь в точке Лагранжа L1 системы Земля-Солнце. Аппарат расположен так, чтобы всегда наблюдать планету с освещенной стороны, съемка ведется с частотой примерно один кадр в два часа. В его задачу входят метеорологические наблюдения и предупреждение о солнечных вспышках.
Почему раз в два часа? Скорость передачи данных низкая для обьема картинки, что он генерирует? Что-то изнашивается на самой обсерватории? Вряд ли там механический затвор. Или просто нет необходимости чаще снимать? Интересует прежде всего, была бы разница принципиальная между двумя часами и 10 минутами например.
Аноним 14/03/20 Суб 18:44:54 55335786
>>553328
>Не надо сотни тонн, вопрос был про маленькие аппараты.
>Стартовая масса будет несколько тонн (ну пусть десять тонн).
>Стратостат вменяемых размеров будет, вероятно, в сотни раз дешевле такого самолёта.
Не будет.
https://ru.wikipedia.org/wiki/Red_Bull_Stratos
Сабж при нагрузке в 1.4т имел объём 850000м3. Это, на минуточку, четыре "Гинденбурга". Конечно, взлетал он слабо заполненным, но на высоте раздувался на максимум, потому что воздух там ОЧЕНЬ разреженный. Даже по сравнению с потолком высотных самолётов. И стоит всё это дохуя, запуск обошёлся в $30млн, что уже сопоставимо с космическими запусками.

Стратостат на 10-15т (это вместе с ракетой очень маленький аппарат, кубсат практически, учитывая, что массовое совершенство у маленьких ракет хуёвое) будет стоить как трижды ёбаная в жопу оверпрайснутая Дельта IV хеви. Не дороговата ли экономия?
Аноним 14/03/20 Суб 19:35:51 55335987
>>553352
а хуй знает, я сейчас историю миссии читаю и её с таким трудом продавили. В один момент заморозили в буквальном смысле, забрали деньги, отменили, но потом восстановили, еще и пилили бюджет на ней знатно, присасываясь к ней чтобы передалать под солнечную обсерваторию, а потом отменяли эти планы, когда деньги уже были потрачены. Её вообще вице президент Эл Гор задумал, может камера и прочее оборудование с тех времён.
Аноним 14/03/20 Суб 19:40:18 55336288
>>553298
>прямо передо мной луна которая освящена лишь на половину
РПЦ не освящает невидимую половину.
Аноним 14/03/20 Суб 19:42:20 55336489
image.png (253Кб, 414x232)
414x232
image.png (3212Кб, 1200x1200)
1200x1200
image.png (904Кб, 1000x666)
1000x666
image.png (3257Кб, 1600x1021)
1600x1021
как будут выглядеть галактики и туманности вблизи? можно ли их увидеть своим глазом так же как и на фото, находясь от них на нужном расстоянии? как туманности выглядят изнутри? небо будет в узорах со всех сторон?
Аноним 14/03/20 Суб 20:46:42 55338090
>>553364
>как будут выглядеть галактики и туманности вблизи? можно ли их увидеть своим глазом так же как и на фото, находясь от них на нужном расстоянии?
Нет, конечно. Даже сам Млечный путь видно только при очень темном небе, а мы находимся прямо внутри него.

Если бы Андромеда была сильно ближе, ее (а не только сам яркий центр) было видно невооруженным глазом как туманное облачко наподобие Магеллановых облаков, но ничего похожего на фотографии с длинной выдержкой увидеть нельзя ни с какого расстояния.
Аноним 14/03/20 Суб 21:16:46 55339391
>>553380
я говорю про наблюдения с космического корабля а не земли
Аноним 14/03/20 Суб 21:40:03 55339892
>>553393
Разница невелика
Аноним 14/03/20 Суб 21:41:25 55339993
>>553362
Вода есть на обеих половинах, так что и освящаются обе
Аноним 14/03/20 Суб 21:50:32 55340094
>>553398
хорошо, но туманности то хоть немного похоже ирл видно будет?
Аноним 14/03/20 Суб 22:13:59 55340295
Может ли экипаж МКС заболеть уханьской простудой?
Аноним 14/03/20 Суб 22:16:35 55340496
>>553399
Чтобы заднюю половину освятить, нужно туда добраться, а там только протестанты и аметисты летали.
Аноним 14/03/20 Суб 22:24:54 55340697
>>553402
Конечно, какой-нибудь турист из Италии мимо проезжать будет, с пересадочкой, заразит там всех.
Аноним 14/03/20 Суб 23:01:47 55340998
>>553399
Задняя половина лунного диска недоступна для освящения. Да и вообще это философско-умозрительный концепт, ведь Луна прибита к небесной сфере, а "с той стороны" сферы может разве что Бог посмотреть. Может там и нет никакой второй половины.
Аноним 15/03/20 Вск 01:50:39 55342699
>>553404
Автохуй. Когда воду в водопроводе освящают, в трубы попа проталкивают штоле?
Аноним 15/03/20 Вск 05:53:01 553430100
>>553398
А если вылететь за пределы млечного и выключить свет на корабле, имея при этом девственно чистые глаза, не испорченные компьютером и вообще?
Аноним 15/03/20 Вск 06:22:31 553432101
Я раньше не задумывался, но загуглив обнаружил такой занимательный эффект как прецессия орбиты луны.
Крутится перегей, а так же независимо с большим сроком узлы. Наклонение я так полагаю статично. Вопрос как загуглить положение узлов и апо\перегея на данный момент?
Аноним 15/03/20 Вск 07:08:43 553435102
>>553432
Если тебе интересны цифры, то воспользуйся вот этим
https://ssd.jpl.nasa.gov/horizons.cgi
хотя там вроде нету узлов прецессии, но может я просто в глаза ебусь. Или тебе нужно что-то очень визуальное?
Аноним 15/03/20 Вск 07:47:32 553436103
>>553435
Ну визуализация никогда не повредит, но если этого нет то это хоть что-то.
Ух а это интерестная штука, не знал что такое есть. Спасибо.
Аноним 15/03/20 Вск 09:25:22 553444104
>>553436
>>553432
Если тебе прям точно прецессию посчитать - делаешь это сам, по актуальным эфемеридам согласно одной из моделей. В астрономии и космонавтике применяется три постоянно обновляемых модели движения тел Солнечной системы:
- JPLовская Development Ephemeris
- EPM от Института прикладной астрономии РАН
- INPOP, модель от Парижской обсерватории
Они немного разные, но расхождения минимальны, если тебе не научную работу писать или траектории АМС/спутников высчитывать детально. Они все учитывают довольно дохуя явлений, от релятивистских эффектов до возмущений от неравномерной гравитации. Опять же, ИПА и IMMCE ведут ежегодники, в которых есть готовые прецизионные расчеты, в том числе и прецессии, но они денех стоят.

Так-то институты ведут и обновляют согласно наблюдениям кучу моделей, о которых сразу не подозреваешь, но которые охренеть как широко применяются. NRLMSISE-00, например - актуальна и в астрономии, и в метеорологии, и в баллистике ИСЗ, и где только не. А обновляют её полтора постдока за еду. Фундаментальные сервисы они такие.
Аноним 15/03/20 Вск 10:02:16 553447105
>>553313
>А почему тебе кажется что луна всегда ровно светит мне неведомо. Возможно по той же причине по которой перевёрнутый вверх ногами или набок текст не осознаётся как перевёрнутый. И вообще это европоцентризм думать что сверху вниз, а не слева на право. Вот тебе флаг Мавритании с изображением луны.
Ну я никогда не видел чтоб луна была под углом. Почему-то в моем петербурге она всегда вертикальная.
Аноним 15/03/20 Вск 12:41:58 553457106
>>552819 (OP)
Насколько сильно фонит Земля на 50 герцах?
Аноним 15/03/20 Вск 12:53:19 553458107
>>553430
Твой глаз в любом случае не имеет такой выдержки чтоб цвета различать. Камерой увидишь, как и на земле. Глазом - никак и никогда.
Аноним 15/03/20 Вск 12:54:20 553459108
Аноним 15/03/20 Вск 23:48:24 553584109
>>553430
На пиках фотографический шоп фантазия автора скомпилированный из видимого, радио, ик, уф, рентгена и других диапазонов. В видимом свете вообще нихера кроме нескольких точек не будет видно, и скорее всего вообще без цветов.
Аноним 16/03/20 Пнд 01:40:26 553600110
>>553457
Практически никак. Не говоря уже о том, что чем меньше частота, тем меньше фонит.
Аноним 16/03/20 Пнд 12:09:09 553621111
>>553584
>>553600
не верю. что вам вообще об этом знать? вы это видели вообще???
Аноним 16/03/20 Пнд 13:38:37 553628112
>>553457
>Насколько сильно фонит Земля на 50 герцах?
>
Вроде в ЛЭПах используется другая частота.
Аноним 16/03/20 Пнд 17:03:28 553637113
поясните пожалуйста, если у вас есть космический аппарат для полёта к другой планете, и его двигатели вывели его на межпланетную переходную орбиту но еле-еле, буквально на десяток-другой метров в секунду скорость превышает вторую космическую. с какого момента такому космическому аппарату можно начинать тормозить чтобы перейти на орбиту планеты назначения если у этого аппарата допустим очень слабые движки ионники или вообще какой нибудь парус и ему нужно много времени чтобы затормозить?
Аноним 16/03/20 Пнд 17:28:25 553639114
>>553637
Если еле-еле, то по большому счёту похуй. От любого чиха орбита сразу превратится в эллиптическую, и дальше уже можно будет вальяжно тормозить где удобнее. Гораздо хуже всё с быстрыми кораблями, с избыточной дельтой в 10-15км/с, а манёвр даже в упрощённой КСПшной механике длится часами.

Общий принцип такой:
1. От пизды ставишь точку манёвра, тянешь требуемую дельту в любом направлении и считаешь длительность манёвра.
2. Охуеваешь от длительности.
3. На траектории ставишь точку назначения, от считываешь от неё точку в 70-80% длительности манёвра. Нужен хороший такой запас.
4. Тормозишь вручную на глаз. Всё это время смотришь, куда смещается траектория, и подруливаешь еблом корабля, чтобы она более-менее оставалась на месте.
Аноним 16/03/20 Пнд 17:44:35 553643115
>>553364>>553380
>ничего похожего на фотографии с длинной выдержкой увидеть нельзя ни с какого расстояния.
Есть один хинт.
Нужно съебать в тропические ебеня с нулевым световым загрязнением и упороть веществ, позаимствованных у шамана местных индейцев. Волшебные кактусы, вот это всё. Если всё сделано правильно, то зрачок расширится сверх нормы, а светочувствительность сетчатки возрастёт. Ну и мозг будет веселее обрабатывать информацию. Тёмные слабоконтрастные объекты будут не фильтроваться, а наоборот дорисовываться.
Плюс дополнительные приёмы с витамином А, красными очками, тренировками и.т.п. Можно очень пиздато забустить ночное зрение. http://lumenhouse.ru/articles/2851/
Аноним 16/03/20 Пнд 18:03:36 553645116
>>553639
Лол, ебать я снайпер. Думал, это в огуречном треде вопрос.
Но ирл всё то же самое, только манёвры будут длиться не часами, а месяцами.
Аноним 16/03/20 Пнд 18:51:12 553647117
>>553645
>Но ирл всё то же самое, только манёвры будут длиться не часами, а месяцами.
У тебя там двигатели Эпштейна чтоль есть, чтобы месяцами топливо жечь можно было?
Аноним 16/03/20 Пнд 18:52:30 553648118
>>553645

к сожалению я ничего не понял из твоего ответа. разве если сразу начать гасить скорость то не получится так что если ты загасишь допустим эти 10-20м/с которые у тебя выше второй космической допустим за день или два с момента старта с орбиты земли, у тебя траектория твоей орбиты разве не замкнётся обратно вокруг земли?
Аноним 16/03/20 Пнд 18:55:47 553651119
насколько вообще эллиптической может быть орбита вокруг земли в солнечной системе? чем это обусловлено? может какоето тело иметь орбиту вокруг земли в солнечной системе с самой дальней точкой в 1 а.е. от земли, например? может 10, 100, 1000? есть предел?
Аноним 16/03/20 Пнд 18:58:16 553652120
>>553651
Сферу Хилла гугли.
Аноним 16/03/20 Пнд 18:59:26 553653121
Аноним 16/03/20 Пнд 20:41:57 553660122
Аноним 16/03/20 Пнд 20:48:27 553661123
image.png (31Кб, 1200x638)
1200x638
простейший способ сделать гору 100 км и в дальнейщем с ее помощью выходить на орбиту. пишу сюда спросить как назвать гору. я и так знаю что это сработает.
Аноним 16/03/20 Пнд 20:49:02 553662124
>>553661
ах да, назвать думаю "атлант расправил плечи" вам как?
Аноним 16/03/20 Пнд 21:19:24 553664125
>>553662

лучше "хмурый атлант одиночества"
Аноним 16/03/20 Пнд 21:24:08 553666126
>>553664
я не под хмурым не надо тут свои смештучки суватьт!
Аноним 16/03/20 Пнд 21:38:24 553670127
>>553661
Делай сразу до геостационарной орбиты, а то еще разгоняться надо.
Аноним 16/03/20 Пнд 21:44:20 553671128
>>553670
А в центре горы ебануть лифт, и получится космический лифт.
Аноним 16/03/20 Пнд 21:45:10 553672129
>>553670

>разгоняться

пушку поставим на вершине горы
Аноним 16/03/20 Пнд 23:34:09 553677130
>>553661
Предлагаю ещё сделать верхушку горы плоской и соответственно саму гору расширить в несколько раз, чтобы было где хорошенько разогнаться. Потому что тебе надо не 100км вверх, а 7.9км/сек вбок.
Ой, а чего это земная кора вдруг провали
Аноним 16/03/20 Пнд 23:46:54 553678131
Аноним 17/03/20 Втр 00:14:31 553681132
>>553651
Орбита - это математическая условность, всего лишь прогноз траектории тела. Если ты этот прогноз тупо по школьному кеплеру будешь считать, то как только тело пересечет точку, в которой гравитация СС уравновешивается гравитацией соседних звездных систем и галактики в целом - оно обратно уже не вернется ни за какие шиши. Но тела летают не как Кеплер приказал, а туда, куда их тянет инерция, другие тела, и прочие внешние силы в данный момент.

Снаружи много звездных систем, они распределены неравномерно и тащат к себе по-разному, да еще и движутся постоянно. Поэтому полость гравитационного влияния Солнечной системы имеет неровную и постоянно меняющуюся границу. А тело с перицентром, почти упирающимся в край этой полости, может на одном витке вернуться назад, а на другом уже потеряться.

Во-вторых, по СС насквозь иногда летают всякие говна, которые могут повлиять на траекторию или захватить что-нибудь по пути, особенно если это что-то сейчас на самом краю гравитационного влияния Солнца и легко сталкивается на другую сторону. Вон например несколько десятков тысяч лет назад через Солнечную систему пролетала звезда Шольца.

Но в первом приближении можно сказать так. Сфера Хилла Солнца в отношении Альфы Центавра (наиболее близкая к нам сейчас) это примерно 2.3 световых года минимум. Поэтому если не тянуть апоцентр дальше примерно пары световых лет, назад тело скорее всего вернется, хоть и через огромное время, при условии что звезды не сдвинутся за это время (а они сдвинутся, и прилично).

Главное не забывать что 1) граница гравитационного влияния точно не известна, и 2) есть куча факторов кроме гравитации, такие как солнечный ветер, магнитное поле и т.п., которые вполне могут стать решающими на самом краешке полости, где гравитационное влияние незначительно.

>>553652
Сфера Хилла определена для пар тел. Эквипотенциальная граница Солнечной системы это сумма всех возможных сфер.
Аноним 17/03/20 Втр 00:19:49 553682133
>>553681
Мне кажется, он имел в виду что-то типа спутника Земли.
Аноним 17/03/20 Втр 00:21:28 553683134
>>553681
Блять, вот это я в шары ебусь!
В любом случае, это все применимо и к Земле - как только ты пересекаешь границу влияния, ты уже обратно не вернешься.
Аноним 17/03/20 Втр 00:21:44 553684135
Аноним 17/03/20 Втр 00:57:13 553687136
Аноним 17/03/20 Втр 02:10:08 553692137
image.png (91Кб, 299x300)
299x300
Аноним 17/03/20 Втр 02:25:09 553694138
1584401108985.jpeg (29Кб, 240x320)
240x320
Аноним 17/03/20 Втр 07:52:38 553699139
Вот вы говорите гора 100 км провалится под кору. Насколько я понял верхние слои подминают жидкую астеносферу и не дают подняться выше. Но что если нагружать породу до тех пор, пока литосфера не встретится с нижней мантией? Будет ли тогда увеличен предел высоты предполагаемой горы?
Аноним 17/03/20 Втр 07:57:03 553700140
>нижняя мантия
твёрдая нижняя часть верхней мантии
>>553699
фикс
Аноним 17/03/20 Втр 08:32:20 553701141
image.png (21Кб, 603x603)
603x603
image.png (220Кб, 448x259)
448x259
двач я придумал вечный двигатель. берешь ионизированный газ, накачиваешь им куб посильней,магнит, и весь газ по магнитным линиям будет упорядоченно двигаться, тем самым давя только на 1 стенку, а значит создавать тягу вперед!! но все же мне думается, что то то подведет. что может подвести?
Аноним 17/03/20 Втр 08:35:53 553705142
>>553701
У тебя стационарное магнитное поле. Тягу ты какую-то получишь только при включении магнита, при изменении поля.
И нет, не получишь в закрытом объеме, поле и газ уравновесятся, постоянную тягу не будешь иметь.

>>553703
>>553702
Заебал бампать на слоудоске.
Аноним 17/03/20 Втр 08:43:39 553709143
>>553705
если это не возможно, тогда как летают летающие машины?
Аноним 17/03/20 Втр 08:53:04 553711144
Аноним 17/03/20 Втр 08:53:15 553712145
image.png (275Кб, 631x300)
631x300
>>553709
С помощью воздушных движителей, пропеллеров, например.
Аноним 17/03/20 Втр 08:57:24 553713146
>>553712
Это тачка?? это самолет...
Аноним 17/03/20 Втр 09:00:24 553714147
image.png (1029Кб, 1000x667)
1000x667
Аноним 17/03/20 Втр 09:01:37 553715148
image.png (111Кб, 275x183)
275x183
>>553714
вот летающая машина
Аноним 17/03/20 Втр 09:03:13 553716149
>>553715
Хватит троллить тупостью.
Аноним 17/03/20 Втр 09:04:32 553717150
>>553716
какой тупостью? самолет с крыльями. машина с колесами. летающая машина летает без крыльев, но с колесами
Аноним 17/03/20 Втр 09:05:52 553718151
>>553717
Такого не бывает, маня.
Аноним 17/03/20 Втр 09:09:39 553719152
Аноним 17/03/20 Втр 10:40:30 553724153
>>553211
>Из-за постоянных ударов метеоритов образуются осколки и пыль


В голос!

>>553149
Неизвестно анон.
Аноним 17/03/20 Втр 10:43:25 553725154
>>553724
>В голос!
Какой голос? У тебя луна кратерами целиком покрыта, кратеры от чего на ней?
Больше так не тупи.
Аноним 17/03/20 Втр 10:47:56 553726155
>>553725
Постоянные == раз в сутки минимум

Мне чот слабо верится, что там дело только в метеорах и Солнечном свете.

Кстати, раз уж ты мне ответил. Если что-нибудь интересное почитать про ползающие камни и там и на Земле? Вообще есть что-нибудь не уровня вау шок, а простые факты, которые подтверждены?
Аноним 17/03/20 Втр 10:51:26 553727156
>>553726
>Мне чот слабо верится, что там дело только в метеорах и Солнечном свете.
Там миллионы лет это все проворачивалось, а не в одночасье.
Кратеры как раз и показывают, что эрозия весьма слабая, т.к. при сильной эрозии их бы замывало. Тем не менее - песочек реголит по сторонам раскидывает только в путь. Алсо термическое воздействие может обрушать стенки кратеров.
Вот и какая-никакая (почти никакая) эрозия.

>Если что-нибудь интересное почитать про ползающие камни и там и на Земле?
То?
>Вообще есть что-нибудь не уровня вау шок, а простые факты, которые подтверждены?
Нет, это зогач. Камни сами по себе не катаются.
Аноним 17/03/20 Втр 10:54:10 553728157
>>553727
А что советские учёные говорят по этому поводу?


И да, тебе же явно минимум 30 лет и ты ни разу не слышал про ползающие камни? Как так?
Аноним 17/03/20 Втр 10:57:02 553729158
>>553728
>А что советские учёные говорят по этому поводу?
Не интересовался.
>И да, тебе же явно минимум 30 лет и ты ни разу не слышал про ползающие камни? Как так?
Но ведь слышал.
Ты спросил "не уровня вау шок, а простые факты, которые подтверждены". Такого нет.
Тот факт, что тебя не устроил ответ подсказывает, что ты можешь быть сильно предвзят и ожидаешь подтверждения своим убеждениям. https://en.wikipedia.org/wiki/Confirmation_bias
Аноним 17/03/20 Втр 10:58:53 553730159
>>553729
Ты куда-то в сторону ушёл. Печально, что этот вопрос не изучали учёные, а ещё удивляются, почему их мало кто слушает.

>Не интересовался.

Вот это странно, они хотя бы точно туда луноходы отправляли.
Аноним 17/03/20 Втр 11:02:28 553731160
>>553730
Зачем заведомо шизоидный вопрос изучать?
Камни не двигаются, если их не двигать, это факт.
Аноним 17/03/20 Втр 11:07:34 553732161
>>553726
На земле есть ползающие камни, да. Это связанно с ветром, как конкретно перенаправляю тебя в гугл. На луне ветра нет, поэтому и ползающих камней там тоже нет.
Аноним 17/03/20 Втр 11:08:53 553733162
>>553732
>Это связанно с ветром
Со льдом.
>The movement of the rocks occurs when large ice sheets a few millimeters thick and floating in an ephemeral winter pond start to break up during sunny days. Frozen during cold winter nights, these thin floating ice panels are driven by wind and shove rocks at speeds up to 5 meters per minute.
Аноним 17/03/20 Втр 11:09:18 553734163
>>553731

А предположения, что это может быть некое природное явление, которое происходит после дождечка в четверг и с камнями, состоящими из кремния строго в диапазоне 60 - 70%, которые в этот особенный день особенно электризуются, так же важна поверхность, которая из-за влажной почвы и некого неизвестного эффекта.

>>553732
>Это связанно с ветром, как конкретно перенаправляю тебя в гугл

Лучше бы ты просто ссылку на видос закинул, в которой явно видно подтверждение таким заявлениям. Иначе как со всей наукой - подсчитали кароч, воооооот, двигаются ветром кароч, вооооот.

>>553733
А вот это уже интересно
Аноним 17/03/20 Втр 14:56:43 553743164
Аноны, а зачем люди сейчас занимаются освоением космоса? На космос тратятся огромные деньги, а выхлоп очень небольшой. Посудите сами: на то, чтобы "прогрессом" доставить партию трусов и водичьки для космонавтов тратятся деньги, на которые можно было бы построить несколько школ. На разработки тратятся огромные деньги, а в итоге всё, что нам доступно - улететь на несколько сотен км от земляшки, чтобы... чтобы нихуя. Люди осваивают космос уже почти 60 лет, но ни НЛО, ни дачи на луне - ничего.
Аноним 17/03/20 Втр 15:14:26 553744165
>>553743
>Аноны, а зачем люди сейчас занимаются освоением космоса?
Потому что больше деваться некуда, раз можем в космос, то будем осваивать космос, файнал фронтир, вот это всё.

>На космос тратятся огромные деньги
Да нет, не такие уж и огромные.

>а выхлоп очень небольшой.
Вот быдлу всегда сразу выхлоп подавай. Наукой занимаемся, за Землёй наблюдаем, за Солнцем, узнаём, что вообще творится, это очень важно. Не летать в космос - это как сидеть в шалаше в лесу и отказываться выглянуть наружу.

>тратятся деньги, на которые можно было бы построить несколько школ
В которых никто ничему не научится, если не будет яркого примера и мотивации, а космос как раз таким примером служит.

>улететь на несколько сотен км от земляшки, чтобы...
Проводить там эксперименты.
Аноним 17/03/20 Втр 16:05:14 553745166
>>553743
Я помню твою пасту, болезный.
На пасту ответ пастой:

Блять больше всего я ненавижу мудаков которые говорят , что космос не нужен.
Я всегда ссу на ебало этим уродам, которых к сожалению огромное количество. Сраные ебаные кретины, тупорылое быдло мечтающее о теплом угле, вкусной еде и мясной дырке. Неужели сложно понять простую вещь - Движение вперед это охуенно. Фронтир, неизведанное, полететь на Марс потому что можем и дальше.
И вот эти чмыри которые ненавидят космос, видят только унылое его использование приходят в спейсач и начинают кукарекать, что Марс не нужен.
Сдохните твари. Все что создано вокруг вас руками человека создано наперекор таким как вы. Все время существования человечества мы волоком тащим вас визжащих,упирающихся еблом по говну, камням и грязи вперед к прогрессу и таки протащили из обезьяны к запускающим в космос корабли разумным существам.
Аноним 17/03/20 Втр 16:20:02 553748167
изображение.png (558Кб, 576x864)
576x864
Что делать, если я хочу быть таким охуенным как пикрел и строить ракеты, но я нищеинженер из рашки и мой максимум - РДТТ с потолком в жалкие ~20 км
Аноним 17/03/20 Втр 16:23:07 553749168
>>553748
Запускай огурцач, обмазывайся РСС и строй йобы скрывая свой бугурт.
Аноним 17/03/20 Втр 16:28:33 553750169
изображение.png (170Кб, 640x360)
640x360
Аноним 17/03/20 Втр 16:31:01 553751170
>>553748
>РДТТ с потолком в жалкие ~20 км

Ракеты на твердом топливе и на орбиту выходят, за ними (а конкретно за японской SS-520) рекорд по минимальной массе орбитальной ракеты — 2,6 тонны.
Аноним 17/03/20 Втр 16:33:50 553752171
Аноним 17/03/20 Втр 16:33:59 553753172
>>553751
>на твердом топливе
смотря каком, если будешь активно интересоваться кое чем, то придет дядя майор и невежливо попросит пройти на бутылку
Аноним 17/03/20 Втр 16:35:57 553754173
>>553752
я уже, но что-то действительно стоящее построить требует оверкилл денег и усилий
Аноним 17/03/20 Втр 16:40:10 553755174
>>553748
Оцени свои финансовые возможности, и если их не хватает на обучение в США, то поступай куда-нибудь в Германию на бюджет, обучайся, работай, зарабатывай денег, продвигайся по службе, потом сможешь открыть свой космический бизнес, к тому времени уже возможно рынок услуг по снабжению Луны будет, можно делать лендеры, платформы всякие, роверы.
Аноним 17/03/20 Втр 16:51:16 553756175
>>553755
ТАм это самое, короновирус и вот это вот всё
Аноним 17/03/20 Втр 16:58:22 553757176
>>553756
Давай постараемся вскрыть эту тему. Ты молодой, шутливый, тебе все легко. Это не эбола и даже не грипп. Сюда можешь лезть. Серьёзно, только старперы и будут жалеть. Лучше не пытайся отмазываться и запомни что тут писалось. Я вполне понимаю, что данным сообщением не вызову дополнительного интереса, но сразу хочу подначить пытливых - вперед. Остальные просто не найдут.
Аноним 17/03/20 Втр 17:57:07 553761177
>>553754
>требует оверкилл усилий
Карьера пикрил - не для тебя
Аноним 17/03/20 Втр 18:15:11 553762178
Аноним 17/03/20 Втр 18:38:29 553764179
Аноним 17/03/20 Втр 19:01:04 553766180
>>553753
Полибутадиен запрещен, что ли, или перхлорат аммония, или порошок алюминиевый?

Это ж не экзотика типа фтор-мочи, такое твердое топливо жгут тысячами тонн каждый год.
Аноним 17/03/20 Втр 19:32:09 553772181
>>553748
Охуенность для обывателей. Будь суровым и неизвестным. Вкатывайся в безопорные технологии создания тяги.
Аноним 17/03/20 Втр 19:35:10 553773182
>>553772
>Вкатывайся в безопорные технологии создания тяги.
Это в зогаче изучением подобных технологий занимаются. У нас на доске все же больше про реально работающие технологии.
Аноним 17/03/20 Втр 19:48:41 553776183
>>553772
>Вкатывайся в безопорные технологии
Случайно прочитал "теологии". На секунду пост показался чуть более осмысленным.
17/03/20 Втр 19:58:33 553778184
>>553766
>
>Полибутадиен запрещен

Где ты его достанешь? А перхлорат аммония не то что бы запрещен, просто на карандаш могут поставить, ибо из него делают бризантные ВВ.
Аноним 17/03/20 Втр 20:00:30 553779185
>>553757
Вообще-то речь про то что европка закрыла границы. Да и из рашки вроде как не выпускают нынче. Поправьте если не прав.
Аноним 17/03/20 Втр 21:38:26 553790186
>>553778
>Где ты его достанешь
Да хоть прямо на али продают HTPB от 1 литра, это ж просто каучук, которым порошки склеивают в гранулы, чтобы они горели медленно, а не сразу по всей своей огромной поверхности, он применяется еще дохуя где.

Комбо перхлората аммония и алюминиевой пудры действительно звучит довольно стремно и может вызвать определенные вопросы, но законом не запрещено. Пока ты не начнешь их смешивать и заливать в формы, доебаться формально и не к чему, в закрытом гараже можно хоть шаттловский SRB построить.
Аноним 17/03/20 Втр 22:15:42 553795187
>>553778
И давно это из него делают бризантные ВВ?
>>553790
>законом не запрещено
Товарищи! Учитесь юриспруденции надлежащим образом!
Аноним 17/03/20 Втр 22:29:38 553797188
>>553744
>>553745
Понятно. Машем хуями в алкогольной яме дрочим вприсядку Исследуем космос! Усилий затрачиваем дохуя, а мкс в итоге выглядит как сраный гараж деда ебинея.
Аноним 17/03/20 Втр 22:32:51 553798189
>>553790
Как только вынесешь из гаража на всеобщее обозрение, у окружающих тут же возникнут вопросы, а ктонибудь очень сознательный и в мусарню позвонит. Разве что класть сразу в закрытый прицеп. И то гаишники могут прошмонать
Аноним 17/03/20 Втр 22:34:01 553799190
>>553790
> не к чему
П Р Е К У Р С О Р Ы
Р
Е
К
У
Р
С
О
Р
Ы
Аноним 17/03/20 Втр 22:45:08 553800191
>>553795
Покажи закон, запрещающий перхлорат аммония или алюминиевый порошок.

>>553799
Прекурсоры запрещены только у наркотических веществ, окислитель и топливо сами по себе не являются взрывчатыми веществами и полностью законны.
Аноним 17/03/20 Втр 22:45:43 553801192
изображение.png (483Кб, 1061x696)
1061x696
Хуле так дорого?
Аноним 17/03/20 Втр 22:47:18 553802193
>>553800
>
>Покажи закон, запрещающий перхлорат аммония или алюминиевый порошок.
Так никто не говорит, что закон запрещает. Просто на тебя внимание обратят. Поставят на карандаш. Ферштейн? А там до бутылки пол шага, учитывая реалии рашки.
Аноним 17/03/20 Втр 22:52:16 553803194
>>553801
По килограмму и более грубого помола всего 650 р/кг на этом сайте.
Аноним 17/03/20 Втр 22:53:30 553804195
>>553802
С таким настроем в рашке вообще ничего делать нельзя, а особенно сидеть на мейлрушной борде.
Аноним 17/03/20 Втр 22:55:37 553805196
будь спок.jpg (116Кб, 300x387)
300x387
>>553798
>Как только вынесешь из гаража на всеобщее обозрение, у окружающих тут же возникнут вопросы, а ктонибудь очень сознательный и в мусарню позвонит. Разве что класть сразу в закрытый прицеп. И то гаишники могут прошмонать
Ну что вы как маленькие.
Называешь ракету ВЛАДИМИР ВЛАДИМИРОВИЧ ПУТИН-1, рисуешь на ней иконы, триколор, 1941-1945 можем повторить, портрет Кадырова и прочие атрибуты лояльности. Человек в погонах видит эти обереги и задумчиво икает, понимая, что если он помешает запуску ВЛАДИМИР ВЛАДИМИРОВИЧ ПУТИН-1, то премии от начальства не получит. А начальству не то чтобы не похуй, но страшно не выслужиться перед своим начальством. И так до бесконечности. Поэтому человек в погонах говорит "ну ладно" голосом Витали и делает вид, что его тут нет.

В ебанутом цирке важно не содержание, важна подача. С правильной подачей хоть ЯРД у себя в гараже запиливай.
Аноним 17/03/20 Втр 22:56:22 553806197
>>553803
всё равно дороха. 300р была бы норм ценой
Аноним 17/03/20 Втр 22:57:58 553807198
>>553804
>а особенно сидеть на мейлрушной борде.
Это можно, но крайне желательно с VPN
Аноним 17/03/20 Втр 23:02:58 553809199
lol3.png (7Кб, 231x176)
231x176
Аноним 17/03/20 Втр 23:06:31 553810200
>>553801
Это не та ли хуйня, что охуительно взрывалась на складах с мощностью как небольшая атомная бомба? Че-то стремно такое дома хранить.
Аноним 17/03/20 Втр 23:08:31 553812201
>>553790
>Да хоть прямо на али продают HTPB от 1 литра
А ссылочку можно? Поискал, не нашел что-то.
Аноним 17/03/20 Втр 23:11:56 553813202
>>553810
Зато смерть будет быстрой.
Аноним 17/03/20 Втр 23:18:50 553814203
>>553813
И смерть еще половины дома.
Аноним 17/03/20 Втр 23:24:40 553815204
>>553812
Под али подразумевалась алибаба, а не алиэкспресс, всякие материалы типа пластиков, металлов и прочей хуйни именно там продают.
Аноним 18/03/20 Срд 00:38:13 553827205
>>553815
Как там покупать? Чёт не понятно
Аноним 18/03/20 Срд 00:41:25 553828206
>>553827
ПОСАН НЕ ПОКУПАЙ У МЕНЯ ТАК БРАТ ОТ КОРОНАВИРУСА УМЕР ХУЛИ ОН ПИЗДИТ ЧТО ЭТО БЕЗОПАСНО ПИДР??!? тебе не надо. Нафиг рисковать так-то?
Аноним 18/03/20 Срд 01:27:44 553830207
>>553828
Удваиваю этого, с топливами на основе перхлората аммония не шутят, можно и взорваться нахуй очень легко, и на бутылку присесть.

Если хочется чего-то запустить, бери проверенную тысячами ракетомоделистов карамель, импульс пожиже, зато живой будешь.
Аноним 18/03/20 Срд 01:48:16 553831208
>>553082
На полярные пофиг откуда.
Аноним 18/03/20 Срд 03:37:56 553838209
>>553701
>тем самым давя только на 1 стенку
Ошибка. Попробуй подучить физику.
Аноним 18/03/20 Срд 08:24:15 553841210
>>553699
Ты как не извращайся, не изменишь того факта, что земля в макро-масштабах представляет собой жидкое тело, поддерживающее свою форму исключительно самогравитацией. Что-то крупнее прыща на теле планеты уминается в эту форму, независимо от того из чего оно состоит.

Конкретно твоя хуйня была давно концептуализирована, только не в виде ебейшей горы, а в виде пусковой петли. https://en.wikipedia.org/wiki/Launch_loop
Аноним 18/03/20 Срд 08:35:37 553842211
The Chain Fount[...].webm (1691Кб, 720x404, 00:00:35)
720x404
>>553841
Как эта хуйня держится в воздухе.
Аноним 18/03/20 Срд 10:07:01 553851212
>>553830
>>553828
Чё бы ему взрываться? Не держи у открытого огня и ниче не взорвется.
Аноним 18/03/20 Срд 10:13:46 553852213
>>553743
Аноны, а зачем люди смотрят рекламу? На рекламу тратятся огромные деньги. Посудите сами: мало того, что стоимость рекламы включена в стоимость товара, то есть без рекламы товар был бы дешевле. На эти деньги можно было бы построить планету из школ, а маркетолухов сделать учителями в этих школах, а их знания по манипулированию и наебыванию, направить в полезное русло. Вдалбливаю пиздюкам с детства, что модно быть не тупой пиздой говноблогерром, а ученым мужем. Потому то чем круче твое открытие, тем чаще студенточки будут делать тебе минет между парами.
Аноним 18/03/20 Срд 10:47:40 553855214
Аноним 18/03/20 Срд 12:49:22 553876215
>>553810
На тех складах взорвалось, внезапно, около 4000ТОНН (4 ТЫСЯЧ ТОНН). Этот милый Антоша не сможет инициировать свой жалкий килограммчик даже на обычный пук-среньк.
Аноним 18/03/20 Срд 13:04:14 553878216
>>553852
>Аноны, а зачем люди смотрят рекламу? На рекламу тратятся огромные деньги. Посудите сами: мало того, что стоимость рекламы включена в стоимость товара, то есть без рекламы товар был бы дешевле. ...

а) Реклама следствие перепроизводства. Единственный способ ее убрать это сделать недопроизводство что бы люди сами разбирали любые калоши так как других нет.

б) Реклама "твой кампутер за год устарел, купи новый" кажется бессмысленной пока не меняешь временную шкалу на 10-20 лет. Внезапно выясняется что прогресс таки есть, и его двигают те хомячки которые ведутся на рекламу.

> Вдалбливаю пиздюкам с детства, что модно быть не тупой пиздой говноблогерром, а ученым мужем. ... студенточки будут делать тебе минет

Одно другому никак не мешает. Но пока силы говноблоггеров с ментача направлены на воспитание тезиса самок-сжеч-ряяя-нидают.
Аноним 18/03/20 Срд 13:49:51 553883217
Аноним 18/03/20 Срд 17:25:09 553959218
>>553851
В сухом виде взрывается от удара и трения.
Аноним 18/03/20 Срд 19:16:11 553976219
>>553807
Так через впн на мейлаче запрещено сидеть. Догадайся почему
Аноним 18/03/20 Срд 19:42:40 553979220
Аноним 18/03/20 Срд 22:11:39 553998221
>>553842
Момент же. Выдергивает с такой скоростью, что по инерции летит еще дальше вверх.
Вроде сам Молд и объяснял.
Аноним 18/03/20 Срд 22:12:17 553999222
Аноним 18/03/20 Срд 23:15:37 554008223
>>553804
>сидеть на мейлрушной борде.
Сидеть то можно где угодно, а вот перед тем как что-то писать надо думать головой.
Аноним 20/03/20 Птн 16:26:40 554146224
1. Если космонавт заболеет коронавирусом на МКС, его оставят там или отправят на землю?
2. Если нужно срочно вернуть МКСников на землю, они ждут ракету или у них всегда пристыкован модуль для срочной эвакуации?
Аноним 20/03/20 Птн 16:27:28 554147225
>>554146
1. Он не заболеет коронавирусом на МКС.
2. Всегда пристыкован.
Аноним 20/03/20 Птн 16:34:36 554149226
>>554146
как он заболеет-то, не контактирует ни с кем
Аноним 20/03/20 Птн 16:35:56 554150227
>>554149
Но ведь бактерии поднимаются до высоты МКС, вдруг в форточку залетит?
Аноним 20/03/20 Птн 17:12:20 554157228
>>554150
Пьяный астранафт решит покурить в форточку и засосёт бактерию, заражённую коронавирусом?
Аноним 20/03/20 Птн 17:35:03 554163229
>>554157
На ночь после перекура забудет форточку закрыть и ему надует вирус, да.
Аноним 20/03/20 Птн 17:35:24 554164230
Аноним 20/03/20 Птн 18:44:25 554169231
>>554146
>1. Если космонавт заболеет коронавирусом на МКС, его оставят там или отправят на землю?
Оставят там.
Аноним 20/03/20 Птн 18:53:24 554171232
>>554169
Чтоб Землю не заразил?
Аноним 20/03/20 Птн 20:09:38 554175233
Почему не запускают мелкие спутники с суперпушек?
Аноним 20/03/20 Птн 20:13:52 554176234
>>554175
Потому, что их распидорасит об воздух.
Аноним 20/03/20 Птн 20:19:16 554177235
>>554176
Почему бы не распидорасить мелкие спутники об воздух с суперпушек?
Аноним 20/03/20 Птн 20:56:26 554182236
15495450211630.jpg (65Кб, 560x407)
560x407
>>554177
Действительно, почему бы и нет.
Аноним 20/03/20 Птн 21:01:49 554184237
>>554175
Не слышал о спутниках на суперпушках.
Аноним 20/03/20 Птн 22:11:01 554187238
Аноним 20/03/20 Птн 22:35:08 554192239
>>554175
Нет таких пушек, которые могли бы выдать требуемые 10+ км/с.
Аноним 20/03/20 Птн 23:28:04 554198240
cd2[1].jpg (14Кб, 261x199)
261x199
>>553671
Что ты делаешь, содомит? Дома люди спят, а я ржу как конь
Аноним 20/03/20 Птн 23:55:03 554200241
>>553672
А почему нельзя катапульту поставить?
(Я спрашиваю серьезно)
Аноним 21/03/20 Суб 00:19:53 554205242
>>554200
Нужна двухступенчатая катапульта.
Аноним 21/03/20 Суб 04:50:52 554209243
>>554200
Так запускаемая часть один хуй сгорать на гиперзвуке будет.
Алсо нужна такая сила, чтобы не только давать ускорение грузу, но и преодолевать это трение атмосферы которая теперь еще и на плечо катапульты работает. Алсо жопа подсказывает, что таких материалов не завезли которые такую силу бы выдерживали.
Аноним 21/03/20 Суб 09:31:55 554219244
>>554200
>>553672
А что если разгонять ракету не пушкой или катапультой, а сделать так, чтобы она сама себя разгоняла?
Аноним 21/03/20 Суб 10:14:08 554224245
>>554219
>А что если разгонять ракету не пушкой или катапультой, а сделать так, чтобы она сама себя разгоняла?
Уходи, тебе тут не рады
Аноним 21/03/20 Суб 10:18:06 554225246
image.png (61Кб, 299x168)
299x168
>>554219
ПИЗДОС. Ракета сама себя разгоняет!? Как тебе такая ахинея вообще в голову взбрела!?
Не, ну вы слышали!?
Аноним 21/03/20 Суб 17:20:09 554264247
>>554200
1. Катапультой нельзя запустить на орбиту, нужен хоть какой-то движок на запускаемом спутнике. Иначе запущенное либо ебнется обратно на Землю, либо улетит из сферы влияния Земли вообще, если хорошо запустить.

2. Сопротивление атмосферы. Тебе надо не 8 км/с достичь катапультой, а гораздо больше, потому что львиная доля импульса потеряется на сопротивлении атмосферы. А сопротивление растет примерно как квадрат скорости, ох щщи. Нагрев от такого сопротивления вряд ли что-то выдержит, не расплавившись.

3. Ускорение. Катапульта должна разгонять от нуля до 8-10-15 км/с на сравнительно коротком отрезке (размеры катапульты). Соответственно, ускорение должно быть чудовищным, мало что такое выдержит.
Аноним 21/03/20 Суб 17:20:32 554265248
Бывают ли атомные реакции без радиации? Чтоб четко продукты деления и хотя бы без ионизирующего излучения?
Аноним 21/03/20 Суб 17:24:38 554267249
>>554265
>атомные реакции без радиации
Радиация - это поток частиц. Чтобы реакция шла, она должна быть энергетически выгодной. Энергия идет на увеличение импульса частиц. Поэтому какой-то поток будет полюбасу.
Аноним 21/03/20 Суб 17:25:40 554268250
Насколько мощно надо заглубляться и насколько утягивать якорь будет космический лифт если мы его типа построим?
Нужны ли какие-то манипуляторы на земле чтобы подтягивать-подпускать "трос" на земле?
Можно ли сделать земную точку на плавучей йоба-платформе?
Аноним 21/03/20 Суб 17:26:24 554269251
>>554267
Да, я имел в виду радиацию в быдло-понимании. Ионизирующее излучение.
Можно ли иметь атомные реакции без ионизирующего излучения?
Аноним 21/03/20 Суб 17:34:29 554270252
>>554269
Вангну, что нет, но анус ставить не буду.
Аноним 21/03/20 Суб 17:36:37 554271253
>>554270
Мы хоть и на спейсаче, но мы не в спасех треде.

магнат анусов набитых на ставках в спасех треде
Аноним 21/03/20 Суб 18:12:19 554272254
>>554269
Нет. Хотя есть реакции поглощения нейтрона - в некоторых из них ионизирующего излучения не выделяется и они тоже ядерные. Но нейтрон должен быть низкоэнергетическим.
Аноним 21/03/20 Суб 18:18:27 554273255
>>554272
Вся фишка в энергетике этих реакций, да? Они преодолевают барьер и могут взаимодействовать с ядрами?
Аноним 21/03/20 Суб 20:46:55 554281256
>>554171
Чтоб не отправлять экипаж домой раньше срока почём зря, и чтоб не заражать других членов экипажа. Там его изолируют, будет на карантине. Не знаю, есть ли у них аппараты ИВЛ, может и есть, но в любом случае, предполагается, что космонавты ребята крепкие, и корону смогут пережить без осложнений. Но это конечно мои спекуляции, реально я не знаю их протоколов.
Аноним 21/03/20 Суб 20:47:39 554282257
Когда на марсе будем, хлопцi?
Аноним 21/03/20 Суб 20:48:05 554283258
image.png (73Кб, 220x220)
220x220
Насколько сильно надо переделать двигатели обычного реактивного самолета чтобы он мог невозбранно летать на Титане заправленный кислородом или тетраоксидом (без учета замены на коррозийно устойчивые деталей топливопровода)? Давление, подачу, что-то поменять? Может структура пламени заставить двигатель делать иной формы?

Вопрос 2 - что будет являться наиболее удобным источником энергии на поверхности Титана?
Аноним 21/03/20 Суб 20:50:10 554284259
>>554282
Схоняй в махаз, возьми батончик, да присядь, хлопец. Бац! Ты сел на марс!
Не раньше 2030, ничего раньше не светит для людей, никаких нормальных тем. Если КНР и США начнут космическую гонку вотпрямщас - то до 2030 может кто и сядет. А так - хуй там был.
Не факт что на луне-то до 2030 будет человек. Возможно, но я не сильно в это верю.
Аноним 21/03/20 Суб 20:54:15 554285260
>>554284
Грустно как-то! Еще эта панда мия появилась блин...
Аноним 21/03/20 Суб 20:54:21 554286261
>>554284
Да даже если начнут сейчас гонку, максимум к середине 30-х осилят. А на Луне наверное будут, почему нет. В 24 вот не факт, но думаю не сильно позже.
Аноним 21/03/20 Суб 20:55:16 554287262
>>554286
А они астронавтов уже сейчас готовить начали?
Аноним 21/03/20 Суб 21:00:25 554288263
>>554287
Для Луны? Да.

Кстати, вот мои вангования: до 2026 облёт людьми Луны, до 2028 высадка. Старшип полетит на орбиту в 2022, с людьми до 2030, тогда же и к Луне. На Марс с людьми в начале 40-х.
Аноним 21/03/20 Суб 21:01:30 554289264
>>554288
Выглидт адекватно. Надеюсь на такой расклад. На свое 40 летие увижу людей на марсе.
Аноним 21/03/20 Суб 21:02:09 554290265
>>554285
>Еще эта панда мия появилась блин
Тот же вирус и в 2003 такой же бугурт вызывал. Человечество, как вы все помните, вымерло с концами. Конец света случился, и нас с вами не стало.
Самое печальное, что ни один из концов света так и не научил пиздоглазых не жрать мышей и масло из канализации, не срать на улице (да, они тоже), и мыть руки.
Чихай в локоть, не трогай лицо, мой руки - так живи и все будет нормально.

>>554286
>Да даже если начнут сейчас гонку, максимум к середине 30-х осилят. А на Луне наверное будут, почему нет. В 24 вот не факт, но думаю не сильно позже.
С китайцами? Может и проиграют. Эти могут не жалеть жизни тайконавтов и ускорить темпы. Если вдруг такой манямирок случится писькомерства, разумеется.
Я потому и писал - если. Как и раньше - к 2050 может быть и будет что. В ближайшую декаду ничего не намечается реального.

Аноним 21/03/20 Суб 21:04:45 554291266
>>554290
>Тот же вирус и в 2003 такой же бугурт вызывал.
Я не про опасность для ЧИЛОВИЧИСТВО, я просто про панику и экономические тормоза, которые замедлили ход истории (history (Geschichte)).
>Самое печальное, что ни один из концов света так и не научил пиздоглазых не жрать мышей и масло из канализации, не срать на улице (да, они тоже), и мыть руки.
Ты что, это же их КУЛЬТУРА.
> В ближайшую декаду ничего не намечается реального.
А луна?
Аноним 21/03/20 Суб 21:11:50 554292267
>>554291
Пфф, это не вирус замедлил, это я сама замедлила, на лыжах.
Паникеры были и тогда.
Просто поток пиздоглазых калосёров в те годы был не такой активный, вот тот штамм коронавируса тогда и имел меньшее распространение.
>А луна?
Я про высадку на Марс говорил.
С Луной... Не думаю. 20% что будет. 2028 ближайший реальный срок и скорей всего словит перенос.
Аноним 21/03/20 Суб 21:22:21 554294268
>>554293
Ты промахнулся, да?
Аноним 21/03/20 Суб 21:22:48 554295269
>>554293
Хуита.
Рассказ о том что вся вселенная это симуляция студента говнокодера, которому лень было ебашить реалистичную гравитацию и он её задал константой лучше.
Там в итоге люди выстроили из галактик надпись на весь монитор ГОВНОКОД
Аноним 21/03/20 Суб 21:26:48 554296270
>>554295
>выстроили
Лол, продолжение что ли есть? Я думал просто говнопаста.
Аноним 21/03/20 Суб 21:29:50 554297271
Поисните, как у Сагана в "Контакте" Б-г умудрился оставить послание в числе пи? Ведь это математическая константа, не зависящая ни от чего вообще. Даже если бы вселенная была пиздец кривая как у Лобачевского, пи (как отношение длины окружности к диаметру в умозрительной евклидовой геометрии) было бы ровно то же самое. Тождество Эйлера, опять же.
Аноним 21/03/20 Суб 21:48:07 554298272
>>554283
>Насколько сильно надо переделать двигатели обычного реактивного самолета чтобы он мог невозбранно летать на Титане
Чуть более чем полностью.
>Вопрос 2 - что будет являться наиболее удобным источником энергии на поверхности Титана?
Ветряки вряд ли, так что ритэги.
Аноним 21/03/20 Суб 21:50:26 554299273
>>554298
>Чуть более чем полностью.
Я вот чувствовал, что разница должна быть от подачи топлива в окислительную смесь от подачи окислителя в топливную, но я не знаю какая.

>Ветряки вряд ли, так что ритэги.
А почему не сразу реакторы? Теплообмен должен позволять.
Аноним 21/03/20 Суб 21:55:09 554300274
Не может быть такого, чтобы такой большой космос был недоступен для исследований. Во всех фантастических мирах типа экспансии или масс эффекта люди находят нечто, что открывает возможности для экспансии космоса. Значит и ирл что-то такое найдут, новый химический элемент, вещество какое то или ещё что. Просто надо тщательно обследовать свою систему, все астероиды и все планеты. Им миллиарды лет, некоторые вообще прибыли из глубин космоса, где то да найдётся что то эдакое.
Аноним 21/03/20 Суб 21:56:05 554301275
>>554300
>Значит
Это ты так примерно почувствовал?
Аноним 21/03/20 Суб 22:03:54 554302276
Аноним 21/03/20 Суб 22:29:26 554304277
>>554300
а-а, понятно, спасибо что рассказал
Аноним 21/03/20 Суб 22:32:35 554305278
>>554299
>А почему не сразу реакторы?
Ритэги компактнее и легче, только и всего. Если есть возможность привезти целый реактор, то почему бы и нет.
Аноним 21/03/20 Суб 23:51:57 554309279
>>554264
>>554209

А если придумать катапульту-стульчак? То есть, запускать со стульчака-катапульты капсулу, "срущую" огнём (ну на каком там топливе взлетают космонавты)? Ну, то есть, как бы объяснить-то... Есть катапульта в форме стульчака (круг с дыркой). Ставим ракету жопой, из которой издаётся пламя, на этот "стульчак". Запускаем катапульту, ну а потом включаем двигатели, ракета ебашит в космос... профит!!!! Экономия топлива+новые ощущения+нанотехнологии.
Аноним 22/03/20 Вск 01:27:51 554314280
>>554305
По частям нельзя привезти? Да и есть же небольшие реакторы. Вон в подлодках же ставят.
Аноним 22/03/20 Вск 01:33:31 554315281
Я не понел, почему катапульту нельзя поднять на аэростатах на 20 км над землёй и запускать оттудова? Алсо запускать можно рейлганами. Вон учёные достигли скорости снаряда в 8 км/с. Это больше первой космической.
Аноним 22/03/20 Вск 02:33:47 554317282
>>554273
Кто они-то блядь?
Аноним 22/03/20 Вск 02:34:42 554318283
>>554295
Не было там про кривую гравитацию
Аноним 22/03/20 Вск 09:21:39 554331284
>>554300
Нет, не найдется. И скажи спасибо, что мы хоть в телескоп можем увидеть что происходило в прошлом. Мир не сахар, никто ничего тебе не должен. Даже с ядерным синтезом вполне может оказаться, что человечество не сможет его использовать, а уж про полеты к звездам за человеческую жизнь лучше вообще не думать
Аноним 22/03/20 Вск 09:25:38 554332285
>>554331
>Даже с ядерным синтезом вполне может оказаться, что человечество не сможет его использовать
Но оно уже полвека как может.
Аноним 22/03/20 Вск 11:59:31 554336286
Может ли быть так чтобы на планете острова были как грибы - с узкой ножкой и широким всем остальным? Необязательно земляные грибы в воде, а вообще в целом?
Аноним 22/03/20 Вск 12:40:24 554340287
>>554332
да и гравитацию ещё до нашей эры использовали
чё там пиздят что не погут её объяснить
Аноним 22/03/20 Вск 12:43:46 554341288
>>554340
Могут же. Уже лет пятсот как могут. Каждый раз точнее, но от изменения объяснения принципы не менются, все работает.
Аноним 22/03/20 Вск 13:39:57 554346289
>>554336
Да, если они изо льда
Аноним 22/03/20 Вск 13:42:19 554347290
image.png (6Кб, 366x243)
366x243
>>554346
Вопрос о том, чтобы ножки касались дна, чтобы внизу было твердое, чтобы вокруг жидкость и грибовидный остров. Пикрил.
Аноним 22/03/20 Вск 14:22:05 554350291
>>554341
Объяснить и описать две большие разницы.
Аноним 22/03/20 Вск 14:26:43 554351292
>>554350
Работает? Работает. Предсказательную силу имеет? Имеет.
Чего ты ещё хочешь-то?
Некоторые вещи могут быть непостижимы с наскоку обычным мясным мозгом который вне мира который изучает.
Кто может чествно похвастать, что ориентируется в 4д пространстве? (Я не про рубик). Кто может похвастать, что понял квантомех на интуитивном уровне?
Аноним 22/03/20 Вск 14:36:55 554352293
>>554347
Есть такая штука дифференциация вещества. Тяжёлое вещество будет стремится вниз, а лёгкое вверх. Не забываем о сопромате, который не даёт образовываться вертикальным структурам из природных материалов выше определённого предела. 15 км для земли например. Подобная структура, если и возможна, то либо на маленьких планетах с не очень большой гравитацией, тогда и жидкость будет явно не вода, а какая-нибудь специфичная штука, что не улетит в космос. Естественно-природно такое образоваться явно не могло. Что на астероиде н-километров было, что-то жидкое занимающее больше 80% объёма. Это уж какая-то дичь. Но если к примеру сделать ножку покороче, эдак на 3 порядка, и пошире раза 4 то сопромат вроде позволит. Но и тогда должны быть определённые условия существующие 1 раз в несколько кластеров сверхскоплений галактик.
Аноним 22/03/20 Вск 14:38:44 554353294
>>554352
А ножка и не обязана быть несущей. Более того, она могла бы быть плавающей, отсутствующей.
По-моему, Пратчетт что-то такое писал, нет?
Аноним 22/03/20 Вск 14:43:59 554354295
>>554351
"Понимание", как и "восприятие" с "интуицией" вообще-то субъективное понятие. Квант мех ими не оперирует.
Описание действительно описывает и действительно если ты прочитаешь про тигра, что у него есть полосы ты действительно придя в зоопарк их увидишь. Эволюционное объяснение же этих полос будет кардинально отличатся.
Аноним 22/03/20 Вск 14:51:44 554355296
>>554353
Насколько я знаю Пратчетт писал фентези про ведьм и смерть с косой. Если ножка зависнет, это значит что она имеет такую же плотность как окружающая её жидкость. И даже если эта ножка состоит из сверхпрочного материала ближайшее небесное тело своими приливными силами разрушит этот материал в щепки(ну допустим километровые щепки) И они в свою очередь плавно растекутся по всей сфере на которой они имеют такую же плотность как и окружающая их среда.
Аноним 22/03/20 Вск 14:58:28 554356297
>>554354
Развей свою мысль, пожалуйста. Пока что ты мне ничего нового не сказал.
Аноним 22/03/20 Вск 15:19:56 554358298
>>554356
Речь изначально была о том, что мы гравитацию описали, но не знаем как её по нормальному использовать ибо не знаем как она устроена. А потом ты что-то про интуицию начал загибать, и я повторил тезис, что объяснение и описание разные вещи. Поясни какую мысль я должен развить?
Аноним 22/03/20 Вск 15:23:03 554359299
>>554358
>но не знаем как её по нормальному использовать ибо не знаем как она устроена
Это не так. Мы имеем хорошее представление как она устроена и используем её нормально.
> потом ты что-то про интуицию начал загибать,
Про вещи которые с тем как окружение устроено не вяжутся.
Аноним 22/03/20 Вск 15:43:45 554369300
>>554359
Ты хочешь сказать ты 100 % уверенно знаешь какие процессы протекают чтобы гравитирующий объект гравитировал?
Какие вещи не вяжутся с окружением? Чьим окружением? Как не вяжутся?
Аноним 22/03/20 Вск 19:10:00 554404301
>>554369
>Ты хочешь сказать ты 100 % уверенно знаешь какие процессы протекают чтобы гравитирующий объект гравитировал?
Разумеется, нет. Наука так не работает.
Я знаю что в 100% случаев на земляшке и близ нее буду иметь ону траекторию.
>Какие вещи не вяжутся с окружением? Чьим окружением? Как не вяжутся?
Никакие. С реальностью не вяжутся.
Аноним 22/03/20 Вск 22:18:52 554424302
>>554314
>Вон в подлодках же ставят.
Мне лень гуглить массовогабаритные параметры реакторов апл, но там явно счёт идёт на тонны и метры. А у ритегов на килограммы и метры. Но тут вопрос в том, какая нужна мощность. Если нужны мегаватты, то юзать ритеги действительно глупо.
Аноним 23/03/20 Пнд 00:20:33 554432303
>>554314
>По частям нельзя привезти?
Нет. Все космические реакторы запускали целиком. И устанавливают реакторы только на заводах
Аноним 23/03/20 Пнд 03:14:08 554445304
Почему наливание кипятка звучит как-то мягче, иначе, нежели наливание не горячей воды?
Аноним 23/03/20 Пнд 04:48:14 554450305
Какие животные лучше всего себя чувствуют в невесомости?
Птицы почему-то вообще нихуя не могут в нее, вроде бы.
Аноним 23/03/20 Пнд 08:02:57 554454306
Анон, вот ходили пару лет назад слухи о планах отправить к центавре армию нано-спутников. Интересно, какой начинкой современная наука может снабдить такие аппараты и какие данные о другой системе эта начинка может в теории собрать и насколько реальна их передача из другой системы обратно на Землю ?
Аноним 23/03/20 Пнд 08:07:43 554455307
>>554454
Обычным консюмер-грейдом в космическом исполнении.
Когда спамишь - можно забить на redundancy, оно у тебя в числе аппаратов заложено.
Передавать будет 4 года, когда и если долетит. Передаст инфу об атмосфере на планетах.
Аноним 23/03/20 Пнд 08:36:29 554456308
>>554450
Чем сложнее устроен организм и больше приспособлен к конкретной среде, тем хуже.
Бактерии проще и способны жить везде им похуй.
На насекомого и так хуй какая гравитация действует.
Мышка всё же норное животное, плавать как космонавты не сможет, но в трубах-террариуме вполне обитают.
Птицы мало того что сложноустроены, так ещё и специалисты противодействия гравитации. Гравитации нет механизмы работают в холостую и им хуёво.
Люди как и любое среднеразмерное животное будет чувствовать себя хуёво, но многие косяки сглаживает универсальность и возможность жить в любых средах за счёт интелекта. У животного с ёба инстинктами, развилась бы паника и разрыв сердца от охуевания.
Аноним 23/03/20 Пнд 08:38:45 554457309
>>554456
>У животного с ёба инстинктами, развилась бы паника и разрыв сердца от охуевания.
Достаточно продвинутый моск как у хомосапенса может корректировать восприятие. Ты падаешь. Но ты достаточно долго падаешь чтобы ты мог собраться с мыслями и что-то делать.
Аноним 23/03/20 Пнд 10:23:25 554459310
>>554455
>консюмер-грейдом в космическом исполнении
Чиво блядь?
Аноним 23/03/20 Пнд 11:29:38 554462311
>>554459
КОНСЮМЕР ГРЕЙДОМ В КОСМИЧЕСКОМ ИСПОЛНЕНИИ
Конкретнее - не специализированные жирные рад-защищенные йобы, а более-менее обычные процы с РТОСом на борту в трипликации. Дешево и многие из них помрут. Но дешево.
Аноним 23/03/20 Пнд 13:05:50 554470312
Аноним 23/03/20 Пнд 13:18:39 554473313
Аноним 23/03/20 Пнд 17:22:29 554484314
>>554450
>Какие животные лучше всего себя чувствуют в невесомости?
Бамп вопросу.
Аноним 23/03/20 Пнд 17:33:29 554485315
.jpg (151Кб, 768x768)
768x768
>>554300
>Значит и ирл что-то такое найдут
Хуле тут искать? Все уже найдено до нас.
Аноним 23/03/20 Пнд 17:36:38 554486316
image.png (298Кб, 480x362)
480x362
>>554485
Но мы пока не видели Лилуминаии Лекатариба Ламина-Тчай Экбат де Себат.
Аноним 23/03/20 Пнд 17:44:16 554487317
.jpg (77Кб, 640x637)
640x637
.webm (188Кб, 1024x435, 00:00:01)
1024x435
>>554486
Вы просто не обращаете внимания.
Аноним 23/03/20 Пнд 17:45:53 554488318
image.png (149Кб, 680x680)
680x680
>>554487
Обращаю. Пятый элемент продается практически в каждом продуктовом этойстраны.
Аноним 23/03/20 Пнд 17:47:58 554489319
image.png (137Кб, 373x210)
373x210
>>554488
>>554487
Пик не рилейтед, кстати, я говорил про бор.
Аноним 23/03/20 Пнд 18:26:20 554493320
>>554484
>>554450
Плохо чувствуют себя те, кто умеет летать, по понятным причинам. Ещё вроде бы кошки плохо переносят, опять же, потому что у них есть программа на случай падения, и вообще они сильно полагаются на вестибулярку, которая там не работает. Лучше всего переносят рыбы, насекомые, одноклеточные.
Аноним 23/03/20 Пнд 18:37:45 554494321
>>554493
>Лучше всего переносят рыбы, насекомые, одноклеточные.
Доставьте инфу по рыбам!
Как у них плавучесть работает-то?
Аноним 23/03/20 Пнд 18:46:15 554495322
>>554494
>Как у них плавучесть работает-то?
Как и на Земле, епта, рыбы имеют нейтральную плавучесть из-за того, что их средняя плотность равна плотности воды, и это соотношение сохраняется при любом ускорении свободного падения. От внезапного пропадания гравитации плавать они не перестанут.
Аноним 23/03/20 Пнд 18:49:12 554496323
>>554495
Эээ... Они ориентируются благодаря плавательному пузырю, который их ориентирует вверх.
Без него они плавают без ориентации.
И другое - я нихуя не могу по рыбам в космосе нормальное найти.
Аноним 23/03/20 Пнд 19:07:20 554497324
Аноним 23/03/20 Пнд 19:10:39 554498325
image.png (75Кб, 500x327)
500x327
>>554497
Вот я дегенерат. Мог бы и сам найти.
Спасибо, анон, добра и здоровья тебе.
Аноним 23/03/20 Пнд 19:14:59 554499326
>>554497
>А. А. Леонов в бортжурнале сделал запись[25]:
>«Как себя чувствуют рыбки? -Хорошо, они все погибли»

Бесчуственные совки!
Аноним 23/03/20 Пнд 19:18:11 554501327
image.png (279Кб, 640x476)
640x476
>>554499
Или наоборот. Сочувствовали и поняли что им лучше погибнуть чем дальше страдать.
Аноним 23/03/20 Пнд 19:29:53 554504328
Народ, вы видели Ланиакею?
вы, сука, понимаете, что каждый сраный атом в видимой вселенной встроен в гравитационную структуру, которая падает на два великий аттрактора?
как у вас пархатый язык поворачивается кукарекать про расширение вселенной после этого?
Аноним 23/03/20 Пнд 19:32:33 554505329
>>554504
Лол.
У тебя с ложки суп упал вниз на тарелку. ЗНАЧИТ ВСЕ ПАДАЕТ В ЭТОМ НАПРАВЛЕНИИ.
Когда как в реальности тебя и твой суп все время тащило к солнцу. А ты не замечал.
И сейчас ты приходишь, отрицаешь все наблюдения и говоришь, что все летит к Аттрактору, забыв про объективную реальность.
Аноним 23/03/20 Пнд 19:33:30 554506330
>>554450
Черви-пидоры и простейшие
Аноним 23/03/20 Пнд 19:33:54 554507331
>>554505
>отрицаешь
какую нахуй объективную реальность?
Аноним 23/03/20 Пнд 19:34:36 554508332
>>554462
А космическое исполнение по-твоему в тернировании?
Аноним 23/03/20 Пнд 19:40:56 554510333
>>554504
А ничего, что мы улетаем от аттрактора в десять раз быстрее, чем падаем на него?
Аноним 23/03/20 Пнд 19:43:22 554511334
>>554510
Если б мы улетали от него, его не называли бы аттрактором. Полетим мы быстро, решительно не скоро.
Аноним 23/03/20 Пнд 19:48:05 554513335
>>554506
А самые сложные кто?
Инб4 люди

>>554508
Нет. Не только, но оно очень важно.
Аноним 23/03/20 Пнд 20:00:31 554518336
>>554513
Всё-таки индустриальное, военное и космические исполнения - это про жырноту транзисторов, а не кратность дублирования. Электронику итак дублируют. Правда, не в одном чипе.
Моё ИМХО - нечего экономить на спичках, процы составляют не такую уж большую долю стоимости научных аппаратов, а корректная их работа критична. Правда, можно съэкономить на энергопотреблении, но тут просто хуй к носу не прикинешь.
Аноним 23/03/20 Пнд 20:05:44 554519337
>>554518
>съэкономить
Ну ебаный ты помидор. Анон, ей-богу.
Аноним 23/03/20 Пнд 20:43:01 554521338
>>554510
тогда бы гравитационная структура развалилась бы миллиарды лет назад
а она существует потому что Аттрактор структурирует материю так же как ЦЧД структурирует галактики
все манятеории давно пора заново пересматривать
Аноним 23/03/20 Пнд 20:44:11 554522339
>>554511
>Полетим мы быстро, решительно не скоро
когда остатки нашей системы будут наматываться на ближайшую ЧД
Аноним 23/03/20 Пнд 20:51:52 554523340
>>554504
>аждый сраный атом в видимой вселенной встроен в гравитационную структуру, которая падает на два великий аттрактора
Боюсь тебя расстраивать, но гравитация не настолько дальнодействующая.
Аноним 23/03/20 Пнд 20:54:15 554524341
>>554519
Блядь, ну смотрите, чему меня тут научили
Аноним 23/03/20 Пнд 22:42:06 554550342
>>554523
объективные данные про Ланиакею говорят обратное
кого мне слушать? ученых астрономов или твой вскукарек на Дваче?
Аноним 24/03/20 Втр 01:21:44 554560343
>>554499
>>554501
Разве это не метафора? Дескать на том свете им хорошо.
Аноним 24/03/20 Втр 01:49:21 554562344
Если все что мы на небе доходит до нас с большим пингом то почему мы видим падающие звезды со скоростью света ирл без задержки?
Аноним 24/03/20 Втр 01:58:19 554563345
>>554562
Задержка все равно есть. Ну и это никакие не звезды, а микрометеориты, до которых от наблюдателя от силы сотня-другая километров, и пинг там идет на микросекунды.
Аноним 24/03/20 Втр 12:59:38 554602346
Как можно получать электроэнергию, какие способы?
Градиент температур с помощью кипятильника ядерного или химического.
Фотовольтаика.
Есть ли какие-то хитровыебанные способы которые из-за сложности или неудобства не юзают, а могли бы?
Аноним 24/03/20 Втр 13:59:22 554607347
>>554602
Можно получать электроэнергию из людей, при этом подключив их к виртуальному миру, чтобы они думали, что они живут как обычные люди, ходят на работу и так далее.
Аноним 24/03/20 Втр 14:01:11 554608348
image.png (1212Кб, 1200x680)
1200x680
>>554607
Пробовали уже, полная хуйня выходит.
Аноним 24/03/20 Втр 14:20:29 554614349
>>554602
Это ты способы преобразования описал, а не источники. Градиент температур обычно преобразуют в электричество через пар, механику и электромагниты, а не напрямую.

Если тебя интересуют готовые аппараты (источник+преобразователь) применительно к космосу, есть например ядерно-оптические преобразователи, например. Сумрачная идея какого-то злого гения из ВНИИЭФ. Частицы делящегося материала подвешены в газе под давлением 100 атмосфер, и возбуждают его атомы распадом, тот начинает светиться. Поскольку это практически монохромное излучение - можно его преобразовывать фотовольтаикой с огромным КПД. Плюс в том, что ты получаешь охуенный источник электричества, по плотности на кг сравнимый с баком бензина, что бесценно для космического аппарата. И предельно простой и не имеющий термодинамических и сложностных проблем реактора при этом, т.к. реакция не идёт. Минус в том, что это натурально грязная бомба. Бак с радиоактивным говном под 100 атмосферами, малейшая протечка в атмосфере и мало не покажется никому. А если ебнет в космосе, лишишься источника энергии (и возможно аппарат распидорасит). Даже если просто стравит, будет менять траекторию реактивной струей. В отличие от РИТЭГа, которому пробоина от маслины на орбите сравнительно похуй.
Аноним 24/03/20 Втр 14:40:56 554622350
>>554614
Впервые слышу, как называется эта штука?
Аноним 24/03/20 Втр 14:46:22 554625351
>>554622
ЯОП, ядерно-оптический преобразователь. Концепт, который давно известен, но вяло исследуют, ибо перспектив мало. Что-то говорили про переработку ОЯТ и космическое применение, но это болтовня, слишком опасная штука.
Аноним 24/03/20 Втр 14:50:58 554627352
>>552819 (OP)
Что будет с моим писюньчиком, если я находясь на орбите черной дыры, попробую его засунуть за горизонт событий?
Аноним 24/03/20 Втр 14:53:14 554628353
Чо там с двигателем из микроволновки? Тяга есть?))
Аноним 24/03/20 Втр 14:55:14 554631354
Снимок.PNG (521Кб, 950x694)
950x694
>>552819 (OP)
Почему я данные не могу передать на кербал?)
Аноним 24/03/20 Втр 14:55:57 554632355
>>554628
После того, как на земле продемонстрировали, что направление тяги никак не зависит от ориентации ведра, уже было понятно, что это никакой не двигатель. Нет, тяги нет.
Аноним 24/03/20 Втр 14:58:29 554633356
>>554627
Никто не знает, но можешь попробовать во имя науки, я разрешаю. Правда ты тоже не узнаешь, тебя спагеттифицирует задолго до горизонта

>>554628
Нет
Аноним 24/03/20 Втр 15:00:52 554634357
>>554633
>тебя спагеттифицирует задолго до горизонта
Врети
Аноним 24/03/20 Втр 15:11:20 554637358
15836112398530.jpg (54Кб, 640x491)
640x491
Хммм...
Чоб спросить такого эдакого, у ученых анонят няш-умняш.
Вот в ютубах показывают, что мол БВ произошел в одной точке. Но так ли это? Как обстоят дела на самом деле? Я вот слышал, что как бы везде бахнуло одновременно.
Аноним 24/03/20 Втр 15:16:26 554638359
>>554637
Сингулярности не существует.
Аноним 24/03/20 Втр 15:46:23 554639360
>>554638
Это как? А пространство бесконечно или замкнуто само на себя? А вещество в ней?
Аноним 24/03/20 Втр 15:56:43 554641361
>>554627
Ну, во-первых, с орбиты ты своим писюном до горизонта событий не дотянешься, потому что стабильные орбиты заканчиваются еще задолго до самого горизонта, на 3/2 шварцшильдовского радиуса.

Во-вторых, горизонт это не какая-то осязаемая поверхность ЧД, никакой видимой границы там нет, и засовывать хуй тоже по факту и некуда.
Это просто воображаемая поверхность, с одной стороны которой вылететь наружу из ЧД еще теоретически возможно, хотя для этого и потребуется фантастическое ускорение, а с другой не поможет уже вообще ничего — там для вылета из черной дыры потребуется двигаться быстрее скорости света, что законы физики запрещают.

В-третьих, если тебя не смущает все выше написанное, и ты все же решил пожертвовать писюном во имя науки, то засунуть-то его под горизонт ты сможешь без труда, как и запрыгнуть туда самостоятельно, а вот обратно высунуть — уже нет.
Аноним 24/03/20 Втр 16:15:12 554642362
>>554641
>а вот обратно высунуть — уже нет
Ну вот залупку я за горизонт все таки умудрился засунуть руками. Что произойдет, когда я попробую его вытащить обратно?
Аноним 24/03/20 Втр 16:28:07 554643363
>>554641
Кстати, раз уж такая хуйня. Значит ли это, что человеческий организм разладится и умрет на орбите вокруг черной дыры, даже сверхмассивной (приливные силы минимальны)? Ведь есть, например, кровообращение, да вообще вся хуйня, обязательная для жизнедеятельности. Если за горизонтом событий нельзя двигаться "наружу" вообще, то вблизи, но снаружи, это тяжелее или нет?
Аноним 24/03/20 Втр 16:36:57 554644364
>>554637
Если взять за основу хаббловскую модель расширения, то всё вокруг. Но не бахнуло, а разуплотнилось. То есть 14ккк лет назад кругом был ад пизды пожарче и плотнее чем в любом солнце, а затем расширялось само пространство. Этот вывод делается на основе того, что скорость расширения пространства всегда была более-менее одинаковой, за исключением первых наносекунд, где она была сильно быстрее. Однако если у расширения пространства есть ускорение, вся модель идёт по пизде.
Аноним 24/03/20 Втр 16:37:34 554645365
>>554633
>тебя спагеттифицирует задолго до горизонта
Это зависит от размера ЧД
Аноним 24/03/20 Втр 16:43:06 554647366
>>554642
Около горизонта ты будешь двигаться в околосветовой скоростью, соответственно для тебя время замедлится почти до нуля. Действия по вытаскиванию залупы для тебя будут равны миллиардам лет для чд, за которые ты давно провалишься сквозь горизонт
Аноним 24/03/20 Втр 16:48:11 554648367
Прохуялишься. Я рядом со своим членом на устойчивой орбите нахожусь. Начинаю свой свисток погружать за горизон событий, ради науки. Что происходит в этот момент? Что я почувстую?
Аноним 24/03/20 Втр 16:50:00 554649368
Аноним 24/03/20 Втр 16:50:42 554650369
>>554642
Атомы хуя под горизонтом расцепятся с атомами хуя, находящимися над ним и он порвется.

Т.к. внутри черной дыры никакое взаимодействие не может распространяться в направлении изнутри наружу, межатомные силы просто перестанут действовать. Это ж просто электромагнитное взаимодействие, переносимое фотонами, без фотонов не будет и связи между атомами.
Аноним 24/03/20 Втр 16:59:45 554651370
15839550308640.jpg (34Кб, 417x350)
417x350
>>554650
>электромагнитное взаимодействие, переносимое фотонами
Лишь фотоны могут что то там переносить? А вдруг есть частицы, которые могут на хую вертеть законы относительности нагибая все известные нам ограничения, и переносить энергию в подпространстве)
Аноним 24/03/20 Втр 17:01:19 554652371
>>554643
Орбит там нет. Вблизи к горизонту событий требуется постоянно ускоряться в направлении наружу от него, чтобы не упасть под горизонт, и чем ближе, тем большее требуется ускорение. Самое близкое расстояние, на котором можно обращаться вокруг ЧД с выключенными двигателями — 3 шваркшильдовских радиуса, ближе этого расстояния стабильных орбит не существует.

https://en.wikipedia.org/wiki/Innermost_stable_circular_orbit
https://en.wikipedia.org/wiki/Photon_sphere

Скорее всего, тебя размажет в паштет еще задолго до самого горизонта, когда мышцы и кости больше не смогут выдерживать перегрузки от работающего двигателя.
Аноним 24/03/20 Втр 17:02:31 554653372
>>554651
>А вдруг есть частицы, которые могут на хую вертеть законы относительности нагибая все известные нам ограничения

Решил украсть мою нобелевку за открытие сверхсветового движения, пидор? Аааа, хуй тебе!
Аноним 24/03/20 Втр 17:10:35 554655373
>>554653
>сверхсветовое движение
А ты допускаешь подобное? Расскажи почему. Оч интересно
Аноним 24/03/20 Втр 18:50:18 554661374
Аноним 24/03/20 Втр 18:58:03 554662375
>>554652
Не обязательно орбита прям впритык. Пусть будет 3 радиуса. Вопрос вот в чем: если тело вращается вокруг СМЧД на стабильной орбите достаточно близко к ЧД, будут ли какие-то трудности в перемещении "внутренностей" по телу, то есть в направлении от дыры? Именно из-за искривления пространства, а не из-за приливных сил (считаем, что ЧД достаточно СМ, чтобы они были маленькими). Например, с кровообращением.
Аноним 24/03/20 Втр 19:17:54 554664376
>>554662
Да вроде нет, проблемы могут быть только из-за приливных сил, а вернее, из-за разницы в силе гравитации между, скажем, головой и ногами. На преодоление этой разницы сердцу нужно будет затратить определенную работу, чтобы поднять кровь из более глубокого гравитационного колодца, но просто так сама гравитация никак не будет ощущаться.
Аноним 24/03/20 Втр 22:19:25 554677377
Видел сайт давно, где размер солнечной системы не условный с большими планетами в несколько диаметров друг от друга, умещающийся в размер картинки, а показывался реальный масштаб, Луна размером с пиксель и можно было скроллить колесом мыши до посинения полчаса от Солнца до Нептунов всяких.
Нет ни у кого, а то найти не могу, может его и нет уже? Или подобное что-нибудь.
Аноним 25/03/20 Срд 00:03:25 554686378
>>554644
понятие пространства времени - попрежнему спекуляция основанная на триллионные долях процента которые происходят из-за того что рядом просрался Джон
Аноним 25/03/20 Срд 02:20:13 554693379
Аноним 25/03/20 Срд 07:32:31 554702380
14598086320020.png (20Кб, 317x397)
317x397
Поясните за орбиту. Почему это возможно в огурцах? И почему когда я давно и пытался повторить орбиту в огурцах у меня получалась хуйня, более близкая к реальности, чем эта?
Аноним 25/03/20 Срд 07:34:57 554703381
>>554702
Это и ИРЛ возможно.
Манёвр Оберта работает.
Аноним 25/03/20 Срд 09:08:11 554706382
>>554627
Эх, а меня лет 5-6 назад ровно за этот же вопрос забанили.
Аноним 25/03/20 Срд 09:10:17 554707383
>>554706
Так лучше же. Тебе стало сразу понятнее, что не надо пиструн в черную дыру пихать. А вот тот анон может и не догадаться без бана и попробует.
Аноним 25/03/20 Срд 09:17:51 554708384
>>554703
Насколько я понял подвох в том что компановка спутников редка Так?
Аноним 25/03/20 Срд 09:18:28 554709385
Аноним 25/03/20 Срд 09:37:37 554711386
>>554707
Я решил что астрономы скрывают секреты межгалактической дрочки и уже на пути к А Стрельца.
Аноним 25/03/20 Срд 09:45:17 554712387
image.png (42Кб, 180x251)
180x251
>>554711
>межгалактической
>дрочить с помощью СМЧД своей же галактики
Значение знаешь?
Аноним 25/03/20 Срд 10:07:29 554713388
>>554712
> своей же галактики
Не стоит вскрывать эту тему
Аноним 25/03/20 Срд 10:38:01 554714389
>>554651
А вдруг тебя каждую ночь ебут гномы-ниггеры пока ты спишь|?
Аноним 25/03/20 Срд 10:39:16 554715390
>>554714
Очень надеюсь на это.
Аноним 25/03/20 Срд 11:46:17 554718391
>>554644
>Однако если у расширения пространства есть ускорение, вся модель идёт по пизде.
С чего бы это?
Аноним 25/03/20 Срд 11:48:08 554719392
>>554718
Так манямирок не рушится.
Аноним 25/03/20 Срд 19:48:44 554733393
я понял как терроформировать марс лол
Аноним 25/03/20 Срд 20:12:49 554734394
Аноним 25/03/20 Срд 20:13:50 554735395
Ребятки, как измеряется гравитация в открытом космосе7 Как уловить разницу в гравитации на орбите земляшки и на орбите урана7 В каких единицах измеряется7
Аноним 25/03/20 Срд 22:11:02 554740396
>>554735
>В каких единицах измеряется7
В ньютонах.
>>554735
>Ребятки, как измеряется гравитация в открытом космосе7
В основном математикой. Ну и акселерометрами.
Аноним 26/03/20 Чтв 06:15:14 554755397
image.png (324Кб, 378x600)
378x600
анонасы, есть ли сайт/приложуха где можно просчитать траекторию движения объекта как созданного тобой, спутник допустим, так и естественного тела, поменяв вектор его движения, например луны или сатурна?

>>554735
гравитация либо есть, либо ты падаешь вместе с опорой, тобишь ее нет. разницы невесомости на уране и земляшке нет, а сила притяжения различна. P=gm
Аноним 26/03/20 Чтв 07:33:43 554757398
>>554755
>есть ли сайт/приложуха где можно просчитать траекторию движения объекта как созданного тобой, спутник допустим, так и естественного тела, поменяв вектор его движения, например луны или сатурна?
Есть Universe Sandbox, там чисто посталкивать планеты, но не космические аппараты.

Чтобы всё вместе это GMAT - универсальный ответ на любой вопрос, не считая всяких STK и астродинамических фреймворков. Можешь в GMAT хоть Кербинскую систему засунуть. Правда 1) придется дохуя вбивать и 2) чтобы именно добавить планете пинок в м/с - придется питоном за API подёргать, по-моему NASA не предусмотрело подобного надругательства над природой в интерфейсе.
Аноним 26/03/20 Чтв 09:16:41 554761399
>>554755
Я, видимо, неправильно сформулировал вопрос
Интересует именно открытый космос. Как в открытом космосе определить степень притяжения к звезде, черной дыре7
Аноним 26/03/20 Чтв 09:21:14 554762400
6ae85e3bc3ef221[...].jpg (798Кб, 2100x2010)
2100x2010
Хочу узнать мнение экспертов по тому, какой способ разгона до околосвета (хотя бы 5%) самый дешёвый, с учётом современных технологий, если учитывать как и стоимость самой ёбы, так и цену за условный разгон 10 тонн на 1 миллипроцент от световой.
Аноним 26/03/20 Чтв 09:23:22 554765401
>>554761
Никак.
Свободное падение, орбита и отсутствие притяжения ничем не отличаются.
Аноним 26/03/20 Чтв 09:43:56 554767402
>>554765
Нипонил.. Но ведь с расстоянием сила тяготения уменьшается?.. И это никак не фиксируется?
Аноним 26/03/20 Чтв 09:50:32 554768403
>>554762
Взрыволет на едренбатонах
Аноним 26/03/20 Чтв 09:50:45 554769404
а
Аноним 26/03/20 Чтв 09:53:54 554770405
Аноним 26/03/20 Чтв 10:10:14 554772406
>>554768
Ну по-хорошему это же пиздец дорого. Каждый раз взрыволёт высирает за собой ядерную бомбу, которая не очень дешёвая, а таких бомб нужны сотни. Даже если мы уберём ебанутые километровые медные полусферы, как в Орионе, все системы охлаждения щита + сам щит + другая радиационная защита будут скорее всего охуенно дорогими.
Аноним 26/03/20 Чтв 10:13:33 554773407
>>553800
ПХА сам по себе может взрываться (и гореть)
Аноним 26/03/20 Чтв 11:08:23 554778408
>>554761
>>554735
Напрямую. Т.е. по ускорению, которое придает эта гравитация другим объектам с заранее известной массой. Принцип эквивалентности гласит, что гравитацию ничем не отличить от ускорения. То есть просто смотришь, как объект с заранее известной массой и скоростью отклоняется от прямой, и выводишь суммарное ускорение свободного падения (а следовательно и гравитация) в данной точке пространства.
>В каких единицах измеряется7
Ускорение свободного падения измеряется в м/с² и является векторной величиной.

>>554765
Принцип эквивалентности это хорошо, но на практике на тебя будет действовать градиент гравитации, который никак его не нарушает и который можно измерить. Либо разницей в ускорении головы и ног, либо кручением аппарата от приливных сил.
Аноним 26/03/20 Чтв 11:43:33 554781409
На какой высоте высотометр становится нинужен и собственно высота становится расстоянием? от чего зависит? Опять ж от гравитации?
Аноним 26/03/20 Чтв 12:17:21 554787410
>>554772
ну так сделай многоразовую ядерную бомбу
Аноним 26/03/20 Чтв 12:33:27 554788411
>>554772
А чего ты хотел? Если б всё было так просто, мы бы давно уже по альдебаранам летали. Забесплатно только гравиманёвры.
Аноним 26/03/20 Чтв 12:46:53 554789412
mayor164735955o[...].jpg (384Кб, 1296x1944)
1296x1944
>>554787
>многоразовую ядерную бомбу
Употребляете?
Аноним 26/03/20 Чтв 13:05:59 554790413
>>554787
>ну так сделай многоразовую ядерную бомбу
>>554789
>Употребляете?

Ну так то Дедал и летает на многоразовых аж термоядерных бомбах. Лазерная инерционная инициация реакции, вот это все.

Другое дело что хотя такую многоразовую термоядерную бомбу и придумали - сделать не получается и когда получится - неизвестно.
Аноним 26/03/20 Чтв 13:43:50 554791414
image.png (5315Кб, 1296x1944)
1296x1944
Аноним 26/03/20 Чтв 13:56:51 554792415
>>554790
>Дедал и летает на многоразовых аж термоядерных бомбах
ВТФ? Обычные термоядерки.
Аноним 26/03/20 Чтв 15:36:05 554799416
>>554792
>Дедал и летает на многоразовых аж термоядерных бомбах
>ВТФ? Обычные термоядерки.
Щито? У дедала все механизмы инициации реакции многоразовые и являются частью корабля, в отличие от "одноразовых" атомных бомб. Расходуемая часть это куски замороженнного топлива.
Аноним 26/03/20 Чтв 16:46:05 554809417
image.png (592Кб, 638x338)
638x338
сап, анчоусы, хочу понять каких размеров должна быть картинка на орбите, чтобы ее было видно не вооруженным глазом и в бинокль?
Аноним 26/03/20 Чтв 17:22:29 554814418
>>554781
Там, где начинается космос, т.е. на линии Кармана. 100 км.
Аноним 26/03/20 Чтв 17:39:30 554816419
Поясните за пространство.
Вот, пространство-время искривляется, ладно. Но откуда берётся энергия для этого искривления? Или пространство можно изгибать типа "бесплатно", энергия не тратится?
И ещё вопросик, чем же отличаются две точки в пространстве?
Строя модель, мы всегда берём координатную сетку, но у Вселенной никакой сетки ведь нету?
Аноним 26/03/20 Чтв 17:41:17 554817420
>>554781
все зависит от системы измерения, а еще высота это и есть расстояние
Аноним 26/03/20 Чтв 17:44:24 554818421
>>554778
Градиент заметен только у компактных массивных объектв либо крупным длинным детектором.
Точечно гравитацию не померить.
Аноним 26/03/20 Чтв 17:46:35 554819422
>>554809
Смотри угловое разрешение глаза и желаемое разрешение картинки, умножай и получай расстояние.
Аноним 26/03/20 Чтв 17:47:16 554820423
Аноним 26/03/20 Чтв 18:15:52 554821424
>>554818
>Градиент заметен только у компактных массивных объектв
Приливная стабилизация на орбите Земли это охуенно важный фактор в проектировании ИСЗ, например, и используется для стабилизации некоторых, и это при не особенно-то больших размерах порядка автомобиля. Один из вариантов станции Freedom вполне предлагался приливно-стабилизируемым, например.
> либо крупным длинным детектором. Точечно гравитацию не померить.
Естественно. Но при не-планковских размерах принципиально возможно. Всё зависит от разнесения и точности акселерометров.
Аноним 26/03/20 Чтв 18:54:16 554826425
>>554821
Акселерометры не покажут ускорение, ты же в свободном падении. Тебе надо весы иметь и сравнивать силу между разными частями станции.
Аноним 27/03/20 Птн 01:05:30 554854426
>>554826
Правильно, акселрометры регистрируют вес
Аноним 27/03/20 Птн 04:40:28 554859427
>>554854
Каким это образом? Акселерометр точечный датчик, без реакции опоры никакого веса в свободном падении ты не обнаружишь.
Аноним 27/03/20 Птн 06:41:54 554860428
Аноним 27/03/20 Птн 10:03:30 554864429
>>554860
достойный аргумент для быдла
Аноним 27/03/20 Птн 15:34:07 554896430
1.png (16Кб, 601x516)
601x516
2.png (16Кб, 601x516)
601x516
Возможно ли сделать телескоп в виде простейшего конусного отражателя от фонарика? Понятно, что если смотреть глазами то изображение звезд будет в виде бублика, но ведь можно будет обработать изображение на компьютере и убрать искажения.
Аноним 27/03/20 Птн 15:41:31 554898431
>>554896
Зеркальный конус выйдет дороже и сложнее обычного параболического зеркала.
Аноним 27/03/20 Птн 15:45:22 554899432
>>554898
Его же даже не обязательно зеркалить, можно использовать эффект полного внутреннего отражения.
Аноним 27/03/20 Птн 15:48:12 554900433
>>554899
Чет мой личный дерьмометр подсказывает, что геометрия не сходится.
Попробуй углы посчитать.
Аноним 27/03/20 Птн 17:15:14 554904434
>>554896
а смысл? все равно это не увеличит количество принимаемого света так как оно считается по площади начала конуса, а переотражения только ухудшат всё
Аноним 27/03/20 Птн 23:50:15 554927435
>>554904
>оно считается по площади начала конуса
Эм, схуяли? Свет-то входит в широкую часть.
Аноним 28/03/20 Суб 00:55:04 554930436
IPkjob-1.png (9608Кб, 5608x3739)
5608x3739
Что тут за скопление в центре?
Аноним 28/03/20 Суб 00:56:51 554931437
IPkjob-1.png (9608Кб, 5608x3739)
5608x3739
Аноним 28/03/20 Суб 01:01:12 554932438
IPkjob-2.png (9608Кб, 5608x3739)
5608x3739
>>554930
дашчан шалит, не даёт пикчу запостить
28/03/20 Суб 03:50:38 554938439
ебучий дашчан не показывал мне пики, всё понятно
Аноним 28/03/20 Суб 06:01:56 554944440
Ученые гавной моченые думали, куда улетела тейя после столкновения с землей?
Аноним 28/03/20 Суб 06:02:56 554945441
>>554932
Вау. Ты на ксяоми фотографировал?
Аноним 28/03/20 Суб 06:13:47 554946442
002.jpg (74Кб, 512x342)
512x342
2gExnXvnpoU.jpg (125Кб, 640x427)
640x427
001.jpg (134Кб, 700x398)
700x398
>>554927
>Свет-то входит в широкую часть.
Ну так широкая часть и есть апертура, за лимит которой не прыгнешь. Проще сделать параболическое зеркало такого же диаметра. Оно будет меньше весить, его проще формовать, и искажений оно практически не даёт.

Я круче придумал. Берём радиотелескоп "кратерного" типа (Аресибо, Тьяньян), покрываем его изнутри маленькими зеркалами на поворотных рамках, как в солнечных электростанциях с нагревателем. Получаем супер-зеркало с адаптивной оптикой диаметром в сотни метров, способное в оптический диапазон, и относительно задёшево. Наведение осуществляется смещением приёмника и синхронизированным доворотом зеркал.
Аноним 28/03/20 Суб 10:17:17 554955443
Аноним 28/03/20 Суб 14:14:01 554990444
>>554946
А кратер нахуя? Ровная поверхность тоже подойдет.
Аноним 28/03/20 Суб 15:00:37 555007445
Аноним 28/03/20 Суб 17:18:13 555020446
1YKWeuj.png (458Кб, 803x732)
803x732
Вопрос 1: Почему звезда Марса в небе выглядит красной, хотя сам Марс жёлтенький? Почему красное смещение осуществляется на участки в сраные миллионы километров, а не за десятки световых лет? В солнечной системе пустота межпланетная какая-то особенная, пыльная и тугая, что даже в оригинале синевато-жёлтый Юпитер с Земли видится бело-красно-орнжевым?
Вопрос 2: Какие наиболее разумные и веские аргументы есть в обозримом научном мире, оспаривающие теорию расширения пространства, но при это объясняющие имеющие оптические феномены?
Аноним 28/03/20 Суб 18:06:33 555023447
>>555020
>Вопрос 1
Потому что свет проходит через воздух.
>Вопрос 2
Никакие.
29/03/20 Вск 05:08:25 555063448
Аноним 29/03/20 Вск 06:39:06 555064449
>>554896
Ты только что ренген-гамма телескопы.
Аноним 29/03/20 Вск 06:43:20 555065450
Вот была Тея и другая протопланета называемая по другому. Почему эту протопланету называют землёй?
Аноним 29/03/20 Вск 09:17:51 555071451
>>555065
Потому чт Воду забрал себе Нептун, Воздух - Юпитер, а Огонь - Солнце
Аноним 29/03/20 Вск 09:39:04 555072452
>>555065
Почему реки от истока до устья называют одним именем, хотя в них впадают другие притоки?
Аноним 29/03/20 Вск 15:39:47 555085453
>>555065
>Почему эту протопланету называют землёй?
Ее называют не из-за того что она протопланета или какие-то предшествующие глобальные события. Она всегла была грязью
Ты живешь на планете ГРЯЗЬ
Так повелось.
Предыдущие события не при делах, название прижилось до понимания астрономии
Аноним 29/03/20 Вск 16:24:25 555087454
>>555020
>сам Марс жёлтенький
Он красный.
>Вопрос 2
Tired light
Аноним 29/03/20 Вск 16:29:56 555088455
>>555087
Красным блещет.
Я завидую.
Я лично никак больше блещущей красноватым точкой его не видел.
А кто-то наблюдал целый диск.
Хотелось бы мне тоже.
Аноним 29/03/20 Вск 19:56:32 555096456
>>555087
>Tired light
ОБОСНУЙ @ ПОЯСНИ
inb4: в гугле забанен
Аноним 29/03/20 Вск 20:14:05 555097457
Аноним 29/03/20 Вск 21:18:13 555104458
>>555087
Усталость света это другое название феномена, но никак не объяснение.
Аноним 30/03/20 Пнд 03:48:22 555126459
https://habr.com/ru/news/t/494516/
> На спутнике связи АМ6 проблемы с охлаждением

Чего? В космосе -273. Какие еще проблемы? Что именно я не понимаю?
Аноним 30/03/20 Пнд 07:09:52 555130460
>>555126
У вакуума нет температуры, а температура тела определяется балансом поглощения, тепловыделения и излучения. Зато есть Солнце, которое шпарит в космосе даже сильнее, чем в полдень на экваторе. Тело нагревается Солнцем, при наличии внуренних источников тепла, вроде электронных компонентов, нагревается изнутри, и достигает той температуры, когда начинает излучать все собственное тепло. Ну то есть, скажем, получает 2000 ватт тепла от Солнца, 500 выделяется изнутри, в итоге температура будет такой, при которой тело будет излучать 2500 ватт. Интенсивность излучения растет в зависимости от четвертой степени температуры. При этом излучение работает отводит тепло намного медленнее, чем конвекция и теплопроводность, поэтому охлаждение космических аппаратов представляет собой проблему.
Но это для общего случая, тут же проблема в перераспределении тепла - компоненты спутника рассчитаны на то, что тепло будет от них отводиться. Грубая аналогия - хотя в комнате холодно и ПеКа не перегревается целиком, но от процессора отвалился кулер.
Аноним 30/03/20 Пнд 10:15:42 555136461
image.png (136Кб, 260x325)
260x325
>>555130
>от процессора отвалился кулер.
Тут дело в том, что ты его не прикрутил в мамку, а приклеил на термопасту.
Забавная аналогия.
Всё верно расписал, ты молодец.
Аноним 30/03/20 Пнд 11:29:02 555139462
>>555126
В космосе напрямую обмениваться теплом не с кем. У спутника на орбите Земли бывает только излучательная теплопередача, на "том конце" которой находятся три объекта:
- Солнце (тысячи градусов, периодически закрывается Землей в зависимости от орбиты, занимает малый градус неба)
- Земля (примерно комнатной температуры в среднем, плюс-минус сапог)
- и фон (очень холодный, несколько кельвин, занимает большую часть неба)
Любая теплопередача идёт в направлении от теплого к холодному, и чем больше разница температур, тем она эффективней. На неё можно повлиять, покрывая спутник специальным покрытием, но Солнце все равно пересиливает всё остальное, прилично шпаря. Поэтому многие спутники имеют жидкостное охлаждение электронных нутрей, через трубки с теплоносителем сбрасывая тепло в радиатор, который отражает большую часть спектра Солнца, но в инфракрасном спектре является черным (и хорошо излучающим). Часто радиатором является тупо часть корпуса, который стерпит нагрев, в отличие от начинки. В более сложных случаях (например МКС) радиаторы представляют собой лепестки наподобие солнечных батарей.
Аноним 30/03/20 Пнд 11:34:36 555140463
image.png (1187Кб, 1024x546)
1024x546
>>555139
>и фон (очень холодный, несколько кельвин, занимает большую часть неба)
Всё небо, всё вокруг без исклчения. Все эти яркие/холодные пятна - сильно СИЛЬНО преувеличены, там флуктуации в долях Кельвина.
Аноним 30/03/20 Пнд 14:24:21 555153464
>>555140
>Всё небо, всё вокруг без исклчения
Ну на низкой орбите приличную часть небосвода загораживает Земля. Вот на ГСО уже небольшую.
Аноним 30/03/20 Пнд 14:26:07 555154465
>>555153
Ну это безусловно верно.
Я лишь говорил про то, что нет у небосвода какого-то места где нуль кельвинов и можно туда охлаждаться. Вся вселенная это минимум 4К без исключения (кроме наших криолабораторий)
Аноним 30/03/20 Пнд 14:37:59 555155466
>>555154
Ясен хуй, я про то и не говорил. А охлаждаться таки есть куда, 4 кельвина это офигенно холодно, если у тебя не йоба-телескоп с матрицей при почти абсолютном нуле, а обычная электроника. Суммарно Солнце в наших краях солнечной системы забарывает фон (с точки зрения термодинамического баланса), но у Солнца полная энергия рассеяна по спектру, а излучает нагретый спутник только в дальнем инфракрасном. За счет этой разницы можно эффективно сбрасывать лишнее тепло. (и даже удерживать водород от выкипания при 20K безо всяких зонтиков, см. EUS)
Аноним 30/03/20 Пнд 14:49:35 555156467
Аноним 30/03/20 Пнд 16:29:33 555177468
15837698922610.png (114Кб, 680x559)
680x559
Я слышал что локхид в 2020 предоставит прототип двигателя будущего, может ли быть так, что нефтяной пиздец начался из-за этого?
Аноним 30/03/20 Пнд 16:30:55 555178469
>>555177
Нет, лягух, вообще никакой связи нет и не может быть.
Аноним 30/03/20 Пнд 16:31:43 555180470
>>555178
А что на счёт двигателя локхида?
Аноним 30/03/20 Пнд 16:32:57 555181471
>>555180
Просто очередная йоба. Локхид их постоянно делает.
Аноним 30/03/20 Пнд 16:33:50 555183472
>>555181
Я просто читал на вики что локхид обещал в 2020 предоставить образец йоба движка на йоба технологии, который можно катать в грузовике, обеспечит город энергией и экологичный.
Аноним 30/03/20 Пнд 16:38:35 555184473
>>555183
А, тот самый, что каждый год последние пять лет обещают что ли? Не движок, а реактор?
Ну продолжай следить, в следующем-то точно сделают.
Аноним 30/03/20 Пнд 16:39:17 555185474
>>555184
Ну лан, я просто думал...
Аноним 30/03/20 Пнд 16:39:53 555186475
>>555185
Бывает. Не стесняйся задавать вопросы. От этого ты хуже не станешь.
Аноним 30/03/20 Пнд 16:41:17 555187476
>>555186
Мы случайно не знакомы?
Аноним 30/03/20 Пнд 17:27:28 555197477
httpss3.amazona[...].jpg (76Кб, 800x601)
800x601
>>555183
Этот реактор похоже все. Случилась классика - очередная "мы тут придумали и сделаем быстро, просто и недорого реактор" уже превратилась в "через пару поколений лабораторных установок мы начнем приближаться к термоядерным параметрам", а реактор вырос до пары десятков метров и пары тысяч тонн.
Планировали в 2017 прототип, в 2022 - коммерческий продукт.
Пикрелейтед их роадмап, T5 начали делать в прошлом году. Потом будут Т6, Т7, и только потом прототип. Теперь это типичный реактор-будет-через-20-лет.
Аноним 30/03/20 Пнд 19:11:14 555213478
>>555197
А есть ли список таких дирижаблей будущего по разным отраслям?
Аноним 30/03/20 Пнд 19:15:30 555214479
>>555197
Вообще-то в прошлом году Локхид объявил постройку новых зданий в сканк воркс ради этой хуйни
Аноним 30/03/20 Пнд 19:21:34 555215480
>>555214
>сканк воркс
Это что-то непристойное?
Аноним 30/03/20 Пнд 19:33:05 555216481
image.png (815Кб, 800x600)
800x600
>>555215
Отдел тайных и передовых разработок у Локхида
Аноним 30/03/20 Пнд 20:31:12 555217482
>>555214
И что? Я же не писал, что проект прекращен. Мало того, я же даже написал, что в прошлом году начали работу над следующей лабораторной установкой - Т5. Но от Т5 до реактора как до Луны, они в принципе подойдут к термоядерным параметрам даже не на следующем а через поколение.
Блин, по их планам от 2014 года, у них уже сегодня должен быть работающий прототип реактора, а они пока только с низкотемпературоной плазмой работать учаться.
Да, весьма вероятно, что они будут дальше работать. Но нужно понимать, что теперь это типичный термоядерный проект, который будет разрабатываться десятилетия с неясными перспективами. Весь апломб, что "мы коммерческая фирма, мы те самые Сканк Воркс, которые умеют решать задачи, и сейчас мы покажем этим государственным грантососам, как надо работать" накрылся пиздой. Нет, ну можно в них верить, но пока они повторяют путь всех предыдущих разработчиков термояда, которые начав с отличной идеи и простого прототипа намертво увязают в болоте, с рекатором-через-двадцать-лет.
Аноним 30/03/20 Пнд 20:41:16 555218483
>>555217
вы находитесь вот здесь ...
Аноним 30/03/20 Пнд 20:53:09 555219484
>>555218
>сканк воркс в 21 веке
>этот мем
Последней 80/20 годнотой SW была стелс-табуретка, скунс сдулся когда ушёл Рич. Не стоит применять это к ним.
Аноним 30/03/20 Пнд 20:55:26 555220485
>>555213
На этом не один многотомник можно написать.
Про космонавтику есть например прекрасный бложик https://falsesteps.wordpress.com/ , автор даже в книгу его оформил
Аноним 30/03/20 Пнд 20:56:16 555221486
>>555219
посмотрим че ты скажешь когда ср-72 выкатят
Аноним 30/03/20 Пнд 20:57:48 555222487
.jpg (99Кб, 480x321)
480x321
>>555215
https://www.amazon.com/dp/0316743003
Читни книжца от бати, много инженерно-управленческих баек узнаешь, заодно и что такое сканк воркс поймешь.
Аноним 30/03/20 Пнд 21:04:25 555223488
Аноним 30/03/20 Пнд 21:36:11 555235489
>>555177
Да, это так.
мимо форсер варпдрайва от локхидов
Аноним 30/03/20 Пнд 21:37:34 555236490
Аноним 30/03/20 Пнд 22:40:38 555245491
>>555187
Я крайне сомневаюсь в такой возможности.
Аноним 30/03/20 Пнд 23:04:07 555251492
>>555155
>нагретый спутник
Но ведь мы его охлаждаем, поэтому не нагретый? Нет ли тут обратных связей в смысле охлаждаем спутник->меньше излучает->больше нагревается->больше излучает?

Точнее не так, такие ОС, конечно, есть, я про смещение точки излучения - с падением температуры спутника он начинает излучать ближе к видимому свету, менее эффективно, так? А при нагреве - наоборот?

Блядь, еще стакан виски и я термодинамическое равновесие!
Аноним 30/03/20 Пнд 23:24:39 555254493
>>555251
>Нет ли тут обратных связей в смысле охлаждаем спутник->меньше излучает->больше нагревается->больше излучает?
В какой-то точке наступит баланс, вот и все. Если у радиатора отводящая поверхность большая, он тебя заморозит, если маленькая - на солнце нагреешься.

>с падением температуры спутника он начинает излучать ближе к видимому свету, менее эффективно, так? А при нагреве - наоборот?
Не так. Горячее - ближе к видимому. Чтобы хотя бы начать излучать в видимом свете, надо докрасна раскалиться, а чтобы конкретно заизлучать - как само Солнце.
Аноним 30/03/20 Пнд 23:42:21 555255494
>>555217
Самая жопа в заявлении - термоядерный реактор в контейнере? Это принципиально невозможно БЕЗ ЕБАНОГО ОБЛУЧЕНИЯ ВСЕГО ВОКРУГ.
В реальности не может быть высокоэнергетических реакций без высокоэнергетических частиц которые хуярят кругом.
Если они пытаются выдумать новый вид защиты от радиации - пусть пытаются, достойное начинание, хотя без бэка элементарной физики.
Аноним 30/03/20 Пнд 23:44:54 555256495
>>555255
Не соглашусь. Синтез возможен и без нейтронов в продуктах реакции, об том и токамаки с ITERами.
Аноним 30/03/20 Пнд 23:46:46 555257496
>>555256
Подробнее про синтез без нейтронов.
Ты же в курсе что там не 1+1 = 2, а дохуища реакций, друг?
Аноним 30/03/20 Пнд 23:51:15 555258497
>>555256
>об том и токамаки с ITERами
Да ты сё
Аноним 31/03/20 Втр 00:02:41 555259498
>>555256
Невозможен. Даже анейтронным реакциям всегда сопутствуют нейтронные и дают приличный поток, пусть и в него уходит небольшая доля энергии реакции, но он есть. Токамаки с итерами - про синтез вообще, а не безнейтронный синтез. Анейтронные реакции поджигаются гораздо сложней, тут бы хоть с обычными выход обеспечить, а ты сразу про гелий-3.

>>555255
Поток-то направленный, можно сфокусировать в мишень. Будет хуярить в регулярно меняемую тряпочку.
Аноним 31/03/20 Втр 00:07:50 555260499
1343566211895.jpg (54Кб, 363x500)
363x500
Аноним 31/03/20 Втр 00:26:58 555264500
>>555259
>Даже анейтронным реакциям всегда сопутствуют нейтронные
11B + H
>Поток-то направленный
Щито-щито?
Аноним 31/03/20 Втр 00:29:38 555265501
>>555259
Какой направленный поток, о чем ты вообще?

При слиянии дейтерия и трития получается нестабильное ядро гелия-5, которое моментально испускает нейтрон в случайном направлении и отдачей отталкивается в противоположном. Никакой направленности там нет, плазма хуярит нейтронами во все стороны.
Аноним 31/03/20 Втр 01:02:21 555268502
>>555259
>Поток-то направленный
Это обоснуй.
Как ты направляешь выход ядерной реакции?
Аноним 31/03/20 Втр 09:46:30 555281503
>>555218
Эх, я обожаю эту фразу. Еще можно написать про престьянина 19 века, не понимающего зачем нужен автомобиль, когда есть лошадь. И про тех, кто ищет отговорки, а кто-то ищет решение. И написать, главное, так можно про что угодно, про реактор, про ЕМ-драйв, про вечный двигатель.
У Маска эта надпись использовалась между, внезапно, результатами. А у SW - между невыполненными обещаниями.
То есть пока выглядит так - в 2011 прыгнул Grasshopper сейчас 2017 год, уже должен был слетать Falcon Heavy, через 5 лет должен лететь StarshipBFR, а список все еще выглядит так:

Вот прыгнет Grasshopper тогда и поговорим
=вы находитесь здесь=
Вот когда полетит их Falcon 1, тогда поговорим.
Вот когда будут клиенты, тогда поговорим.
Вот когда будет контракт с NASA, тогда поговорим.
Вот когда полетит их Falcon 9, тогда поговорим.
Вот когда построят свой корабль, тогда поговорим.
Вот когда долетит он до МКС, тогда поговорим.
Вот когда будут запускать геостационарные спутники, тогда поговорим.
Вот когда придумают, как сажать ракеты, тогда поговорим.
Вот когда посадят хоть одну, тогда поговорим.
Вот когда посадят на баржу, тогда поговорим.
Вот когда начнут сажать после вывода на ГПО, тогда поговорим.
Вот когда полетит б/у ракета, тогда поговорим.
Вот когда запустят свой Falcon Heavy, тогда поговорим.
Вот когда вернут обтекатель, тогда поговорим.
Вот когда сделают свой Crew Dragon, тогда поговорим.
Вот когда повторно используют обтекатель, тогда поговорим.
Вот когда начнут возить людей, тогда поговорим.
Вот когда научатся сажать корабль без парашютов, тогда поговорим.
Вот когда запустят корабль на Марс, тогда поговорим.
Вот когда он сядет на Марсе, тогда поговорим.
Вот когда сделают марсианский транспорт, тогда поговорим.
Вот когда высадят людей на Марсе, тогда поговорим.
Вот когда колонизируют Марс, тогда поговорим.
Вот когда терраформируют Марс, тогда поговорим.
Аноним 31/03/20 Втр 10:03:35 555282504
возможно ли выйти в открытый космос в трусах? чтобы из тебя не вытянуло кислород нужно просто заткнуть уши, жеппу, может быть на глаза надеть очки как у дайверов хз нос и зажать рот и, по идее, это ничем не отличиться от обычной задержки дыхания. радиация, излучения лубая другая хуйня ничего тебе за эти 3 минуты не сделает. я прав?
Аноним 31/03/20 Втр 10:44:45 555285505
>>555281
Сектант не палится
Аноним 31/03/20 Втр 10:45:48 555286506
Space Suit Test[...].webm (2507Кб, 426x240, 00:03:27)
426x240
>>555282
Зажать нос и рот ты не сможешь вообще, слишком сложно в вакууме. Через ~15 секунд ты потеряешь сознание, получишь синяки и гематомы по всему телу, начнется декомпрессионная болезнь. После ~40 остановка сердца и всякие необратимые последствия. В целом, считается что больше 90 секунд после резкой разгерметизации не выжить. В 1966 году у NASA техник в вакуумной камере разгерметизировался при испытаниях скафандра, еле успели вытащить. Это классический случай.
Аноним 31/03/20 Втр 10:47:11 555288507
Аноним 31/03/20 Втр 11:19:56 555292508
>>555286
ты описал то, что будет с человеком без задержки дыхания, но этого не произойдет в моем случае. тут кислород будет в легких и кровь будет забираиь его пополняя его, а значит кислород к мозгу поступать будет и значит сознание не потеряется.
Аноним 31/03/20 Втр 11:24:50 555293509
>>555292
Ты не удержившь воздух в легких, у тебя гортань не герметичный клапан. Он способен держать воздух при разнице давлении вверх, когда снаружи больше давления, но вниз не держит, и сразу весь воздух из легких стравит.
Предлагаю построить вакуумную камеру и потестить наши гипотезы на китайцах.
Аноним 31/03/20 Втр 11:35:26 555295510
>>555293
бля это вообще не проблема для этого хоть мини скафандр можно построить в виде маски даже китайцы не нужны
Аноним 31/03/20 Втр 11:37:27 555297511
>>555295
Не мешай моим планам геноцида китайцев, сволота!
Аноним 31/03/20 Втр 12:23:05 555302512
Аноним 31/03/20 Втр 13:07:35 555305513
>>555292
>ты описал то, что будет с человеком без задержки дыхания
Не, я описал что будет при резкой декомпрессии до вакуума. Задерживай или не задерживай, эта хуйня тебя убьет гораздо быстрее чем отсутствие кислорода
Аноним 31/03/20 Втр 13:33:02 555311514
>>555285
Что, у нас теперь есть секта "неверующих во всесилие Сканк Воркс?"
Аноним 31/03/20 Втр 13:47:24 555318515
>>555311
ХЗ. Но верить в маняобещания не надо. Неудивительно, что за веру в неподтвержденные маняобещания записывают в сектанты.

другой анон
Аноним 31/03/20 Втр 13:57:05 555323516
>>555318
>Неудивительно, что за веру в неподтвержденные маняобещания записывают в сектанты.
Ващет наоборот, тут сейчас меня пытаются записать в секту за отсутствие веры в маняобещания.
Аноним 31/03/20 Втр 14:08:56 555324517
>>555305
что убьет то? перепад в 1 атмосферу?
Аноним 31/03/20 Втр 14:17:03 555326518
Аноним 31/03/20 Втр 15:28:14 555333519
image.png (132Кб, 300x164)
300x164
Аноним 31/03/20 Втр 16:40:01 555340520
KODA-Комиксы-др[...].gif (845Кб, 245x218)
245x218
15852318323940.jpg (1303Кб, 2800x1983)
2800x1983
Вопрос по астероиду Didymos B

Вы тут в спейсаче уже знаете его реальную траекторию или это пока не известно?

Может ли так быть, что NASA хочет нанести по нему удар не в качестве тренировки, а потому что эти великолепные 160м в диаметре собираются ебнуть по нашей уютной планете?

Аноним 31/03/20 Втр 16:42:52 555341521
>>552819 (OP)
>Пик4
Почему "греков" и "троянцев" много, а "хильдов" меньше? (Или по массе эти группы одинаковые?) Логика подсказывает, что хильдов должно быть столько же сколько гомеровских астероидов.
Аноним 31/03/20 Втр 16:46:27 555342522
>>552819 (OP)
>Пик1
Если я еще не заебал.
А нет ли примерно такого же, но в виде гифки или вебки. Вот эти вот все ваши гравитационные ямы в движении очень завораживают
Аноним 31/03/20 Втр 17:02:35 555346523
>>555340
С учетом того, что это спутник астероида диаметром порядка 800 метров, а расстояние между ними около 1,2 километра, то нет. Потому как если ебнут, то ебнут оба, и 160 метров Дидимуна будут представлять из себя меньшую из проблем.
А так, в ближайшие 100 лет не будет приближаться ближе 7 миллионов километров.
Аноним 31/03/20 Втр 18:33:45 555372524
15852617747960.jpg (95Кб, 640x640)
640x640
>>555346
>800 метров,
0_0 ух бля

>спутник
Аааа, теперь понятно, почему именно этот. Можно будет точно высчитать эффект воздействия насовской хуйни, да?

Круто, спасибо
Аноним 31/03/20 Втр 18:40:57 555374525
>>555346
Где же организации по защите диких астероидов?
Аноним 31/03/20 Втр 18:43:07 555375526
>>555324
Да. Это оче дохуя, в частности для твоей кровеносной, лимфатической и прочих жидкостных систем.
Аноним 31/03/20 Втр 19:34:46 555379527
Аноним 01/04/20 Срд 05:28:02 555420528
>>555305
Пиздец ты реально тупой.
Поройся на ютубе. Я может ссылку принесу потом, там один перец на пальцах обьясняет, от чего ты сдохнешь в космосе и это не от перепада давления и кровькишкираспидорасило.
В барокамерах с космонавтами уже случалось подобное и нихуя никого не разорвало.
Аноним 01/04/20 Срд 05:32:57 555421529
>>555375
На глубине 10м, давление 2атм, всплывая быстро на поверхность, даже кессонку не получишь. Перепад 1 атм.
Почему вы блять не можете в гугл суки? Это реально тред тупых, не вопросов, а просто тупых.
Аноним 01/04/20 Срд 05:48:05 555422530
>>555282
>возможно ли выйти в открытый космос в трусах?
Будешь замерзать из-за быстрого испарения влаги с поверхности кожи.

>нос и зажать рот и, по идее, это ничем не отличиться от обычной задержки дыхания
Именно так.

>радиация, излучения лубая другая хуйня ничего тебе за эти 3 минуты не сделает
За 3 минуты не успеют. Умрёшь ты не от радиации, не от перепада давления, твою тушку не распидорасит как в кино со Шварцнеггером.

Самое интересное начнётся, когда ты уже не сможешь задерживать дыхание и сделаешь вдох или выдох, то есть твой организм попытается дышать. Сразу же из лёгких улетучится весь объём газа, они станут абсолютно пустыми.
Так как давление в твоей кровеносной системе выше чем снаружи, то все газы содержащиеся в крови, кислород, азот, углекислый газ, начнут проникать из крови через альвеолы в лёгкие. У тебя изо рта повалит пар, разумеется потому что в выдыхаемых газах будет содержаться вода в том числе. Получится обратный процесс, не насыщение крови газами, а дегазация крови. Примерно через 10 секунд ты потеряешь сознание, примерно через 30 секунд, сердце прогонит всю кровь по кровеносной системе и в ней не останется ни какого кислорода, азота, углекислого газа и ты умрёшь. Через три часа твоя тушка остынет настолько что полностью промёрзнет, вся не испарившаяся вода, превратится в лёд.
Аноним 01/04/20 Срд 10:12:24 555427531
15837037452480.jpg (79Кб, 800x450)
800x450
>>555420
Я тебе дал именно то видео про барокамеру с космонавтами, о котором ты говоришь, блять! И цитирую выводы насы из репорта об инциденте.

>>555421
Почему ты не можешь в мозг? При всплытии ты доходишь до нормального давления. Да блять, это тред тупых по ходу. Декомпрессия до вакуума принципиально отличается от декомпрессии до атмосферного давления.

>>555422
>когда ты уже не сможешь задерживать дыхание
Ты его не сможешь сдерживать вообще, уже при сколько-нибудь заметном падении, не то что в вакууме.
Аноним 01/04/20 Срд 12:05:38 555438532
Правильно ли я понимаю что из-за кризиса/короновируса америка нихуево так просядет финансово и полет на луну придется откладывать на 2030-2040 годы?
Аноним 01/04/20 Срд 18:22:21 555475533
>>555422
> Именно так.
Разве не разорвет легкие к хуям от попытки задержать дыхание ?
Аноним 02/04/20 Чтв 01:03:24 555521534
atlante.jpg (50Кб, 500x674)
500x674
Настройки X
Ответить в тред X
15000 [S]
Макс объем: 40Mб, макс кол-во файлов: 4
Кликни/брось файл/ctrl-v
Стикеры X
Избранное / Топ тредов